Site Loader

Содержание

Поэлементное сложение векторов внутри столбца списка



У меня есть этот тиббл, у которого есть столбец списка с векторами в них

df <- data_frame(grp = c("A", "A", "B", "B"),
                 x = rep(c(list(c(1,2,3)), list(c(4,5,6))), 2))

То, что я хотел бы сделать (предпочтительно в tidyverse), — это выполнить поэлементное сложение векторов внутри списков, по существу:

c(1,2,3) +  c(4,5,6) 
# [1] 5 7 9

Этот:

 # A tibble: 4 × 2
 grp         x
 <chr>    <list>
 A        list(c(1,2,3))
 A        list(c(4,5,6))
 B        list(c(1,2,3))
 B        list(c(4,5,6))

Становится:

 # A tibble: 2 × 2
 grp         y
 <chr>    <list>
 A        list(c(5,7,9))
 B        list(c(5,7,9))

Что может быть хорошим подходом?

r dplyr
Поделиться Источник jenswirf
    06 декабря 2016 в 11:07

2 ответа


  • Поэлементное сложение двух списков разной длины?

    Как выполнить поэлементное сложение списков разной длины? Предположим, что 0 для отсутствующих элементов. Примечание: len (a) всегда будет меньше или равен len(b) пример: a=[1,2,3] b=[1,2,3,4,5] Желаемый результат: result=[2,4,6,4,5] Я почти могу, но не совсем могу сделать это с помощью:…

  • Поэлементное добавление 2 списков?

    Теперь у меня есть: list1 = [1, 2, 3] list2 = [4, 5, 6] Я хотел бы иметь: [1, 2, 3] + + + [4, 5, 6] || || || [5, 7, 9] Просто поэлементное сложение двух списков. Я, конечно, могу повторить эти два списка, но я не хочу этого делать. Каков самый Пифонический способ сделать это?



3

Следующее также должно дать вам то, что вам нужно:

dff %>% group_by(grp) %>%
        summarise(x = list(Reduce("+",x))) %>%
        ungroup()

Надеюсь, это поможет.

Поделиться Abdou     06 декабря 2016 в 11:29



2

Мы можем попробовать

library(dplyr)
library(tidyr)
r1 <- lengths(df$x)[1]

unnest(df) %>%
      group_by(grp) %>%
      mutate(grp1 = rep(seq(r1), 2)) %>% 
      group_by(grp1, add = TRUE) %>%
      summarise(x = sum(x)) %>% 
      group_by(grp) %>% 
      summarise(x= list(x))
# A tibble: 2 × 2
#   grp         x
#  <chr>    <list>
#1     A <dbl [3]>
#2     B <dbl [3]>

Поделиться akrun     06 декабря 2016 в 11:14


Похожие вопросы:


Поэлементное сложение разреженного матричного вектора scipy с широковещанием

Я пытаюсь выяснить, как лучше всего выполнить поэлементное сложение (и вычитание) разреженной матрицы и разреженного вектора. Я нашел этот трюк на SO: mat = sp.csc_matrix([[1,0,0],[0,1,0],[0,0,1]])…


R: назначение векторов из списка векторов в качестве значения столбца во фрейме данных из списка фреймов данных

У меня есть два списка одинаковой длины: один-список фреймов данных, другой-список векторов, так что длина каждого вектора совпадает с количеством строк в соответствующем фрейме данных первого…


Как выполнить сложение векторов в Ruby?

Как выполнить сложение векторов в Ruby так, чтобы [100, 100] + [2, 3] доходность [102, 103] (вместо объединения двух массивов)? Или это может быть и другой оператор, например [100, 100] @ [2, 3] или…


Поэлементное сложение двух списков разной длины?

Как выполнить поэлементное сложение списков разной длины? Предположим, что 0 для отсутствующих элементов. Примечание: len (a) всегда будет меньше или равен len(b) пример: a=[1,2,3] b=[1,2,3,4,5]…


Поэлементное добавление 2 списков?

Теперь у меня есть: list1 = [1, 2, 3] list2 = [4, 5, 6] Я хотел бы иметь: [1, 2, 3] + + + [4, 5, 6] || || || [5, 7, 9] Просто поэлементное сложение двух списков. Я, конечно, могу повторить эти два…


CUDA: сложение векторов и размер векторов

Я измерил пропускную способность при выполнении kernel, делая сложение между двумя векторами : __global__ void add(float *a, float *b, float *c, int n) { int tid = blockIdx.x*blockDim.x +…


Поэлементное матричное сложение на выходе cellfun в Matlab

У меня есть функция d2l_dgamma_dgammaT , которая выводит матрицы 6×6. Я хочу выполнить поэлементное сложение этих матриц без использования for loop ниже. A_gamma_temp-это массив ячеек, содержащий…


Objective-C-поэлементное сложение (и деление) двух матриц с Accelerate.framwork

У меня есть две матрицы (типа float * ), так что я думаю, что их можно рассматривать как массивы. Мне нужно сделать поэлементное сложение двух матриц, и я хотел бы максимально увеличить…


R, операции между подмножеством векторов списка

Я уверен, что об этом уже спрашивали, но после долгих поисков я все еще застрял… как вычислить поэлементное среднее значение подмножества векторов списка? список выглядит следующим образом:…


PyTorch поэлементное произведение векторов / матриц / тензоров

В PyTorch, как я могу получить поэлементное произведение двух векторов / матриц / тензоров? Для гуглеров этот продукт также известен как: Продукт Адамара Продукт Schur Продукт Entrywise

Урок 12: Векторы — 100urokov.ru

План урока:

Понятие вектора

Равенство векторов

Сложение векторов

Свойства сложения

Вычитание векторов

Умножение вектора на число

Решение задач с помощью векторов

 

Понятие вектора

Рассмотрим простейшую задачу. Корабль, двигатель которого развивает скорость 20 км/ч, плывет по течению реки, при этом скорость течения составляет 2 км/ч. Какова скорость корабля относительно берега? Очевидно, в данном случае надо сложить скорость течения и собственную скорость корабля:

20 км/ч + 2 км/ч = 22 км/ч

Теперь посмотрим на почти такую же задачу, которая отличается лишь тем, что корабль плывет уже против течения. Для ее решения скорости уже придется вычитать:

20 км/ч — 2 км/ч = 18 км/ч

Получается, что ответ задачи во многом зависит не только от величин скоростей, но и от их направления. Возможны и более сложные случаи, когда корабль двигается на воде перпендикулярно течению или, например, под углом в 60°. Величины, при операции с которыми необходимо учитывать их направление, называют векторными величинами, или просто векторами.

Помимо скорости к ним относят ускорение, силу, импульс, напряженность магнитного и электрического поля и многие другие величины. Те же величины, для которых нельзя указать направление, называют скалярными величинами. Это масса, температура, плотность и т. п. Для выполнения действий с векторами необходимо разработать общие правила их сложения, вычитания, умножения, которые будут справедливы независимо от физической природы векторных величин. И разработать эти правила помогает как раз геометрия.

Для начала введем понятие вектора. Любой отрезок имеет два конца, которые обычно не отличают друг от друга. Однако если одну из этих точек считать началом отрезка, а другую – собственно концом, то у отрезка появится направление. В таком случае его можно считать вектором.

Часто вектора называют направленными отрезками. Обозначают их с помощью стрелок.

На этом рисунке показан вектор, начало которого находится в точке А, а конец – в точке В. При записи в формулах сначала пишут букву, означающую начало вектора, потом обозначение его конца, а над этими двумя буквами ставят стрелочку:

С практической точки зрения приходится вводить в рассмотрение особый нулевой вектор. У него начало и конец совпадают, то есть он представляет собой всего лишь одну точку:

Нулевой вектор необходим, так как нам необходимо научиться выполнять действия над векторами. Мы знаем, что в обычной алгебре используется число ноль. В векторной же алгебре аналогом нуля является как раз нулевой вектор.

Каждый вектор имеет свою длину, которая равна расстоянию между его началом и концом. То есть, если его начало находится в точке А, а конец в точке В, то длина вектора будет совпадать с длиной отрезка АВ. Обозначают длину с помощью вертикальных скобок:

Естественно, что длина нулевого вектора равна нулю.

 

Задание. Найдите модуль вектора, изображенного на рисунке:

Решение. Легко выполнить построение, при котором вектор окажется гипотенузой в прямоугольном треугольнике

Тогда длину вектора можно найти по теореме Пифагора:

 

Равенство векторов

Через начало и конец векторов можно провести прямую. В связи с этим можно ввести понятие коллинеарных векторов.

На рисунке коллинеарны вектора а и b, так как они лежат на одной прямой. Также коллинеарны с и d, так как они лежат на параллельных прямых. А вот вектора и неколлинеарны, так как они лежат на пересекающихся прямых.

Для пары коллинеарных векторов можно определить, являются ли они сонаправленными или противоположно направленными.

Для обозначения сонаправленных векторов используется символ «⇈», а для противоположно направленных «⇅». Можно сформулировать две очевидных теоремы о коллинеарных векторах.

Проиллюстрируем эти правила с помощью рисунка:

Особняком стоит нулевой вектор. Он представляет собой точку, а потому не имеет определенного направления. Поэтому условно его считают сонаправленным с любым другим вектором.

Теперь мы можем дать определение равенству векторов.

 

Задание. Найдите на картинке равные вектора.

Решение. Здесь равны вектора а, и e. Они сонаправлены и имеют длину 6. Вектор с сонаправлен с ними, но его длина составляет только 5 клеток. Длина вектора d составляет 6 клеток, но он не сонаправлен с другими векторами. Наконец, вектор m также не сонаправлен с другими векторами и даже не коллинеарен им.

Ответ: a, и e.

Если началом вектора является некоторая точка А, то можно сказать, что вектор отложен от точки А. Докажем важное утверждение:

Доказать его можно построением. Пусть есть вектор а и точка М. Проведем через М прямую p, параллельную вектору а. Такая прямая будет единственной. Если точка М и вектор лежат на одной прямой, то в качестве прямой p возьмем именно эту прямую. Далее от точки М можно отложить отрезки МN и МN’, длина которых будет совпадать с длиной вектора а. В результате получится два вектора,MN и MN’, один из которых будет сонаправлен с а, а другой – противоположно направленный.

Часто равные вектора, отложенные от разных точек, обозначают одной буквой. Можно считать, что это один и тот же вектор, просто приложенный к разным точкам.

 

Задание. АВСD – параллелограмм, диагонали которого пересекаются в точке О. Определите, равны ли вектора:

Решение.

а) Отрезки АВ и DC равны, ведь это противоположные стороны параллелограмма, по той же причине эти отрезки параллельны. Видно, что они сонаправлены, значит, вектора равны.

б) Отрезки ВС и DA параллельны и равны, но эти вектора противоположно направлены, поэтому вектора НЕ равны друг другу.

в) Точка пересечения диагоналей параллелограмма делит их пополам, поэтому длины отрезков АО и ОС одинаковы. Вектора АО и ОС лежат на одной прямой, то есть они коллинеарны. При этом они ещё и сонаправлены, поэтому АО и ОС – равные векторы.

г) Вектора АС и BD лежат на пересекающихся прямых, то есть они не коллинеарны. Этого уже достаточно, чтобы считать их НЕ равными друг другу.

Ответ: а) д; б) нет; в) да; г) нет.

 

Сложение векторов

Пусть некоторый объект сначала находился в точке А, а потом переместился в точку В. Тогда его перемещение удобно обозначить с помощью вектора АВ. Далее пусть этот объект из точки В переместился в другую точку С.

С одной точки зрения, объект совершил сразу два перемещения, из А в В и из В в С, которые можно представить векторами:

Этот пример подсказывает нам универсальное правило, с помощью которого можно складывать вектора. Его называют правилом треугольника.

С помощью правила треугольника удобно складывать вектора, если конец одного из них совпадает с началом другого. Но что делать, если это не так? В этом случае достаточно от конца одного вектора отложить вектор, равный второму:

 

Задание. На рисунке показаны два вектора. Постройте в тетради их сумму и найдите длину получившегося вектора.

Решение. Перенесем вектор b к концу вектора а. Далее по правилу треугольника на удастся найти их сумму (обозначим этот вектор буквой с):

Теперь найдем длину получившегося вектора. Он является гипотенузой в прямоугольном треугольнике, причем длины катетов в этом треугольнике можно определить по рисунку, они составляют 4 и 6. Тогда длину гипотенузы можно найти по теореме Пифагора:

Отдельно рассмотрим случаи, когда складываются коллинеарные вектора. В этом случае получающаяся сумма окажется коллинеарной каждому слагаемому. Если вектора сонаправлены, то их длина итогового вектора окажется равной сумме длин складываемых векторов:

Если складываются противоположно направленные вектора, то длина их суммы окажется разностью длин складываемых векторов.

Именно по этой причине при решении простейших задач на движение корабля по реке скорость корабля и скорость течения либо складывают, либо вычитают. Дело в том, что в этих задачах складываются вектора скоростей корабля и течения. Когда судно плывет по течению, эти векторы сонаправлены, а когда плавание идет против течения, векторы оказываются противоположно направленными.

 

Задание. Корабль развивает в неподвижной воде скорость 12 км/ч. Он плывет по реке, скорость воды в которой составляет 5 км/ч. Найдите скорость корабля относительно берега, если:

а) судно плывет по течению;

б) судно плывет против течения;

в) судно плывет перпендикулярно течению.

Решение. Во всех случаях итоговая скорость судна является векторной суммой собственной скорости судна и течения реки:

Однако направления этих векторов различны. Найдем решение графически, с помощью построений. В первом случае вектора по условию сонаправлены:

Приложив другу к другу отрезки длиной 12 и 5, получим отрезок длиной 17. Это значит, что в первом случае скорость корабля относительно берега составит 17 км/ч.

Во втором случае вектора уже окажутся противоположно направленными:

Отрезок, соответствующий итоговой скорости, здесь уже равен 7 клеткам, значит, итоговая скорость составляет 7 км/ч.

В третьем случае вектора скоростей перпендикулярны:

При построении получился прямоугольный треугольник, вектор итоговой скорости в нем оказался в роли гипотенузы. Найти его длину можно по теореме Пифагора, ведь катеты нам известны:

 

Свойства сложения

Действия с векторами во многом подобны действиям с обычными числами. Напомним, что в алгебре при прибавлении к числу нуля оно не менялось:

a + 0 = a

Аналогично и при прибавлении к вектору нулевого вектора он не изменится:

Работает ли это правило с векторами? Оказывается, что да. Убедиться в этом можно, построив параллелограмм, сторонами которого являются складываемые векторы:

Видно, что диагональ параллелограмма является суммой векторов, которые соответствуют нижней и крайней правой его стороне. Они обозначены как векторы и b, причем в данном случае к а прибавляется b. Но одновременно эта же диагональ – это сумма векторов, которые соответствуют крайней левой и его верхней стороне. Напомним, что противоположные стороны параллелограмма равны и параллельны, поэтому они и обозначены одним вектором. В этом случае уже к прибавляется a. Результат при этом получается одинаковый, поэтому можно записать, что

На этом примере мы увидели, как работает ещё одно правило сложения векторов, который называется правилом параллелограмма. Если есть два вектора, которые необходимо сложить, то можно отложить их от одной точки, а потом достроить получившуюся фигуру до параллелограмма.

 

Задание. Сложите с помощью правила параллелограмма вектора, изображенные на рисунке:

Решение. Надо всего лишь построить параллелограмм, как показано на рисунке. Его диагональ и окажется искомым вектором:

Ещё один закон, использующийся в алгебре, называется сочетательным законом, записывается он так:

Оказывается, что и при действиях с векторами он также работает, то есть справедливо соотношение:

Здесь оранжевый вектор – это сумма красного (а) и синего (b) вектора. Если к оранжевому вектору добавить зеленый (с), то получится фиолетовый вектор, который, таким образом, является суммой

Желтый вектор – это сумма синего и зеленого вектора. Видно, что фиолетовый вектор представляет собой сумму красного и желтого, то есть он представляет сумму

Складывать можно любое количество векторов. В этом случае надо последовательно прикладывать эти вектора друг к другу, выстраивая «цепочку» векторов. Например, сложение 4 векторов, показанных на рисунке, будет осуществляться следующим образом:

Этот способ сложения векторов именуют правилом многоугольника. Естественно, в силу переместительного закона вектора можно прикладывать друг к другу в разной последовательности, при этом результат будет получаться один и тот же.

 

Задание. Сложите, используя правило многоугольника, вектора, изображенные на рисунке. Выполните сложение двумя разными способами:

В первом случае последовательно сложим вектора a, b, c и d. Во втором случае изменим последовательность сложения. Например, сложим их в порядке d, b, c, a:

Видно, что каждый из двух способов дал один и тот же результат, что ещё раз подтверждает справедливость переместительного закона сложения векторов.

 

Вычитание векторов

Напомним, что в алгебре операция вычитания вводится как операция обратная сложению. То есть если для трех чисел верно соотношение

a + b = c

то разностью чисел с и a как раз окажется b:

c — a = b

Аналогично вычитание понимается и в векторной алгебре. Пусть построены вектора а, b и c так, что

Этот пример показывает, как строить разность двух векторов. На рисунке вектора с и отложены от одной точки, а вектор b, являющийся их разницей, проведен от конца вычитаемого вектора к концу уменьшаемого вектора.

В данном случае под уменьшаемым вектором понимается тот, который в разнице стоит перед знаком минус, а вычитаемый вектор – тот, который находится уже после этого знака. Например, в записи

Вектор а – уменьшаемый, а вектор b – вычитаемый.

 

Задание. Постройте в тетради разность векторов, изображенных на рисунке:

Решение. Заметим, что в условии не сказано, какой вектор из какого надо вычитать. Поэтому можно построить сразу два ответа:

Несложно заметить, две получившиеся разности представляют собой противоположно направленные векторы одной длины. Такие векторы называются противоположными.

Очевидно, что если сложить друг с другом два противоположных вектора, то получится нулевой вектор:

Противоположные вектора играют в векторной алгебре такую же роль, как и противоположные числа. С их помощью удобно выполнять вычитание векторов. Напомним, что для обычных чисел справедливо соотношение:

Поэтому операцию вычитания можно заменить операцией сложения, если вместо вычитаемого вектора взять вектор, противоположный ему. Рассмотрим этот способ на примере. Пусть из надо вычесть b:

На первом шаге надо построить вектор, противоположный b:

Теперь надо просто сложить a и (– b):

В итоге нам удалось построить разность векторов а и b.

 

Умножение вектора на число

Предположим, что нам надо сложить два равных вектора. В результате мы получим новый вектор, который будет сонаправлен с исходным, но его длина будет вдвое больше. Логично считать, что получившийся вектор вдвое больше исходного, то есть он получился при умножении вектора на число 2:

Аналогично можно построить вектора, которые больше исходного не в 2, а в 3,4 и т. д. раз:

Итак, чтобы умножить вектор на положительное число k, надо построить сонаправленный с ним вектор, длина которого в k раз больше.А как умножать вектор на отрицательное число? Здесь нужно использовать противоположный вектор. Логично считать, что он получается при умножении (– 1) на вектор. Зная это, легко умножать вектор и на другие отрицательные числа:

Естественно, что если вектор умножается на ноль, то в результате получается нулевой вектор.

 

Задание. На рисунке показаны вектора а и b. Найдите вектора

Решение. Для построения снам надо сначала умножить исходные вектора на 4 и 2, а далее полученные результаты сложить:

Для нахождения вектора d надо построить вектор, противоположный вектору 2b, и уже его складывать с 4a:

Наконец, для нахождения вектора е необходимо построить противоположный вектор уже для :

Некоторые правила обычной алгебры, касающиеся операции умножения, справедливы и для векторов. Первый такое правило – это сочетательный закон:

Видно, что мы можем либо сразу умножить вектор а на число 12, либо сначала его умножить на 4, а потом на 3. Результат операции при этом не изменится.

Также в отношении операции умножения векторов на число справедлив распределительный закона, которые позволяют раскрывать скобки:

Например, пусть нам надо сложить вектора и . Распределительный закон говорит, что мы можем поступить двумя способами. В первом случае мы просто строим вектора 2а и 3а и складываем их. Во втором случае мы складываем только числа 2 и 3 (получаем 5), и далее уже умножаем вектор а на число 5:

Есть ещё один распределительный закон, в котором в скобках находится уже сумма векторов, а не чисел:

Этот закон можно применить в случае, когда нам необходимо, например, сложить вектора и 4b. Конечно, можно просто построить их и сложить, однако закон говорит, что мы можем сначала сложить aи b, и уже потом эту сумму умножить на 4:

Сформулированные нами законы сложения и умножения векторов позволяют выполнять действия с векторами так же, как с числами. В том числе можно упрощать выражения, содержащие векторные величины. Например, пусть известны вектора а, b и с, и надо найти вектор

Видно, что выражение значительно упростилось.

 

Решение задач с помощью векторов

Вектора активно используются в физике при решении многих задач, однако они также помогают доказывать геометрические теоремы. Рассмотрим несколько примеров, и начнем со вспомогательной задачи.

 

Задание. Известно, что С – это середина отрезка АВ. Докажите, что для любой точки О выполняется равенство:

Используя правило треугольника, вектор ОС можно представить в виде двух различных сумм:

Проанализируем выражение в скобках. Вектора АС и ВС коллинеарны, ведь они лежат на одной прямой АВ. При этом они противоположно направлены. Длина у них одинакова, ведь С – середина АВ. Тогда по определению АС и ВС – противоположные вектора, и их сумма равна нулю:

 

Задание. Докажите, что если в трапеции провести прямую, проходящую через середины ее оснований, то она также пройдет через точку, в которой пересекаются продолжения боковых сторон трапеции.

Решение. Построим трапецию, обозначим ее вершины и середины оснований:

Здесь ABCD – трапеция, основаниями которой являются отрезки ВС и AD. M и N – их середины. Прямые АВ и CD пересекаются в точке O. Необходимо доказать, что прямая MN также проходит через О.

Заметим, что ∆ОВС и ∆ОАD подобны. Действительно, у них есть общий ∠ВОС, а ∠ОВС и ∠ОАD одинаковы как односторонние углы при секущей АВ, поэтому треугольники подобны по 1-ому признаку. Обозначим коэффициент подобия буквой k, тогда можно записать, что

Так как отрезки ОА и АВ лежат на одной прямой, то вектора ОА и АВ коллинеарны и притом сонаправлены, поэтому в (1) отрезки можно заменить векторами:

(это соотношение мы доказали в предыдущей, вспомогательной задаче).

Аналогичную формулу можно составить и для второго основания и его середины N:

Полученное нами равенство означает, что вектора ON и ОМ коллинеарны, а значит, лежат на одной прямой (эти вектора не могут лежать на параллельных прямых, так как имеют общую точку О). Тогда получается, что О, M и N лежат на одной прямой, ч. т. д.

 

[PDF] Зачеты по геометрии. 9 класс

Download Зачеты по геометрии. 9 класс…

БОУ города Омска «Лицей № 64»

Зачеты по геометрии 9 класс Учебник Геометрия 7-9 Л.С.Атанасян

Майслер Елена Вильгельмовна учитель математики

ФИО автора: Майслер Елена Вильгельмовна Полное название учебного учреждения: Бюджетное общеобразовательное учреждение города Омска «Лицей № 64» Полное название разработки: Зачеты по геометрии. 9 класс Предмет: геометрия Класс: 9 Темы, в рамках которой рекомендовано использование заданий: 1. Векторы 2. Метод координат 3. Скалярное произведение векторов 4. Длина окружности и площадь круга Вид контроля: тематический Среда, в которой выполнена разработка: Microsoft PowerPoint Минимальные технические требования: наличие программы Microsoft PowerPoint 2003 Инструментальные средства, которые были использованы при создании заданий программа Microsoft PowerPoint 2003, текстовый редактор Microsoft Word.

Краткая инструкция для обучающихся 1. Внимательно прочти вопросы к зачету. 2. Запиши ответы к вопросам зачета на листочке. 3. Задачи к зачету разбиты на 3 группы: задачи на «3», задачи на «4», задачи на «5». Выбери задачу из группы, которая тебе по силам, и реши ее. 4. В случае благополучного решения выбранной задачи, можешь улучшить свой результат и выбрать задачу из другой группы, оцениваемой более высокой отметкой. 5. Помни! Чем больше решишь задач, тем больше получишь отметок за зачет.

Содержание 1. Векторы Вопросы для самоподготовки

2. Метод координат Вопросы для самоподготовки

3. Скалярное произведение векторов Вопросы для самоподготовки

4. Длина окружности и площадь круга Вопросы для самоподготовки

1. Векторы зачет

2. Метод координат зачет

3. Скалярное произведение векторов зачет

4. Длина окружности и площадь круга зачет

Вопросы для самоподготовки Зачет № 1

9 класс

«Векторы». 1. Определение вектора. Начало и конец вектора. 2. Как отложить вектор, равный данному, от точки. 3. Определение коллинеарных векторов. 4. Сонаправленные и противоположно направленные векторы. Сделать рисунок.. 5. Определение равных векторов. 6. Сложение векторов по правилу треугольника. Сделать рисунок. 7 Сложение векторов по правилу параллелограмма. Сделать рисунок.. 8. Определение разности векторов.. 9. Как выполняется вычитание векторов. Сделать рисунок.. 10. Законы сложения векторов (2 закона). 11. Определение произведения вектора на число. 12. Теорема о средней линии треугольника

Вопросы для самоподготовки Зачет № 2

9 класс

«Метод координат». 1. Теорема о разложении вектора по двум неколлинеарным векторам. 2. Лемма о коллинеарных векторах. 3. Координаты суммы вектров.. 4. Координаты разности векторов. 5. Формула координат вектора по координатам его начала и конца. 6. Координаты произведения вектора на число 7. Вычисление длины вектора по его. Координатам. 8. Координаты середины отрезка. 9. Формула расстояния между точками 10. Уравнение окружности (2 формулы) 11. Уравнение прямой.

Вопросы для самоподготовки Зачет № 3 9 класс «Скалярное произведение векторов» 1. Определение скалярного произведения векторов 2. Свойства скалярного произведения векторов 3. Косинус угла между векторами 4. Скалярное произведение перпендикулярных векторов 5. Скалярное произведение векторов в координатах. 6. Основное тригонометрическое тождество 7. Формулы приведения. 8.Теорема о площади треугольника. 9. Теорема косинусов. 10.Формулы приведения. 11. Теорема синусов 12. Косинус угла между векторами.

Вопросы для самоподготовки Зачет № 4 9 класс «Длина окружности и площадь круга». 1. Определение правильного многоугольника. 2. Формула для вычисления угла правильного n – угольника. 3. Определение описанной окружности. 4. Определение вписанной окружности. 5. Теорема об окружности, описанной около правильного многоугольника. 6. Теорема об окружности, вписанной в правильный многоугольник. 7. Формулы для вычисления площади правильного многоугольника, его стороны и радиуса вписанной окружности. 8. Длина окружности. Длина дуги окружности. 9. Площадь круга. Площадь сектора.

Зачет № 1 1 вариант

«Векторы» Вопросы к зачету

1. Определение вектора. Начало и конец вектора. 2. Сонаправленные и противоположно направленные векторы. Сделать рисунок 3. Сложение векторов по правилу параллелограмма. Сделать рисунок. 4. Свойства умножения вектора на число. 5. Определение разности векторов. 6. Как выполняется вычитание векторов. Сделать рисунок. 7. Теорема о средней линии треугольника..

Задачи

1. В прямоугольнике ABCD AB=2см, BC = 4см, Найдите длины векторов AB, BC, DC 2. Начертите 2 вектора, имеющие равные длины и неколлинеарные. 1. В прямоугольной трапеции один из углов равен 3.120 Начертите два вектора, 0. Найдите среднююимеющие линию, равные если длины меньшая и сонапраленнные.. и большая ABCD боковаяAB=6см, сторона BC=8см, трапеции 1. диагональ В прямоугольнике 4.равны Начертите 8см. векторы Найдите длину вектораAB, AC CD, EF так, чтобы АВ, СD, EF были коллинеарны и /AB/ /CD/ /EF/а 2.Из концов диаметра АВ= 1см, данной окружности 2.Боковые стороны трапеции 13 см и =2см, 15 см, = 3см проведены перпендикуляры АА1 и ВВ к периметр равен 48см. Найдите среднюю линию 1 5. Выполнить вычитание векторов а касательной не перпендикулярной к диаметру АВ. трапеции. = 16см. 3. Найдите НайдитеВВ диаметр окружности, егов концы 1, если АА 1 =7см, а АВ если 6.3.Выполнить сложение векторов трапеция d 15см и 7см.. Дана равнобедренная АВСD. удалены от некоторой касательной на с проведенный из вершины B к 4. Перпендикуляр, В прямоугольнике ABCD АВ=8см, ВС=3см, М7. Сложить векторы большему основанию АD, делит это основание середина стороны АВ. Найдите длину вектора МС.на 2 m nравностороннего из которых треугольника равен 5 см. Найдите 5. отрезка, Сторонабольший АВС p среднюю линию трапеции. равна 6см. Найдите / AB + BC / q

9 класс 2 вариант

1. Определение коллинеарных векторов. 2. Определение равных векторов. 3. Как отложить вектор, равный данному от данной точки. 4. Сложение векторов по правилу треугольника. Сделать рисунок. 5. Законы сложения векторов (2 закона) 6. Определение умножения вектора на число. 7. Теорема о средней линии треугольника.

Задачи 1.1. ВВ прямоугольной прямоугольнике трапеции АВСD один АВ=6см, из углов ВС=2см. равен Найдите 1200. Найдите длинысреднюю векторовлинию АВ, ВС, трапеции, DС. если 2.меньшая Начертите диагональ 2 вектора, и большая имеющие сторона равныетрапеции длины и неколлинеарные.. равны 6 см. 3.2.Начертите Из концов 2диаметра вектора, СD имеющие даннойравные окружности длины и сонапрвленные. проведены перпендикуляры СС1 и DD1 к 4. Начертите векторы не перпендикулярной AB,АВСD CD, EF так, к диаметру чтобы AB, CD. 1.касательной, В прямоугольнике АВ=12см, ВС=5см. CD, Найдите EF были DD1, коллинеарны если СС1 АС. = 13см, и /AB/=3см, а CD= 29см. /CD/=4 см, Найдите длину вектора /EF/=5см. Дана равнобедренная трапеция17см АВСD. 2.3. Боковые стороны трапеции и 13 см, а 5. Перпендикуляр, Выполните вычитание проведенный векторов. из вершины а в Влинию к периметр равен 58см. Найдите среднюю большему основанию АD, делит это основание на 2 трапеции. 6. Выполните больший сложение из которых векторов. равен С9 см.его Найдите d концы 3.отрезка, Найдите диаметр окружности, если среднюю трапеции. удалены отлинию некоторой касательной на 13см и 8см.. 7. Сложить векторы m АВ=6см, n 4. В прямоугольнике АВСD ВC=4см, М – середина стороны АВ. Найдите длину вектора МС. q p 5. Cторона равностороннего треугольника АВС равна 8см. Найдите /AB + BC/

Зачет № 2 1 вариант

«Метод координат» Вопросы к зачету

1. Теорема о разложении вектора по двум неколлинеарным векторам. 2. Координаты разности векторов. 3. Формула координат вектора, если известны координаты его начала и конца. 4. Формула для вычисления длины вектора по его координатам. 5. Уравнение окружности (2 формулы) 6. Уравнение прямой.

Задачи 1. координаты m = a –сb, диаметром если 1. Найдите Напишите уравнениевектора окружности a {7;3}, b {8;0}. МN, если М(-3;-5), N(-7;-3). 2. Найдите длину вектора АВ, АВС если равнобедренный АВ {8;6}. 2.Докажите, что треугольник 3. Найдитеего координаты m = a В(-2;4), + b, если и найдите периметр,вектора если А(-4;1), С(0;1). a [-2;7}, b {1;-3}. 3. Докажите, что четырехугольник ABCD – 4. Точка М середина отрезка АВ. Найдите если A(4;1), прямоугольник и найдите его периметр, координаты точки М, если А(1;-4), В(7;-2). B(3;5), C(-1;4), D(0;0) 5. Найдите координаты вектора АВ, если А(0;3), В(6;6). 1. Найдите длину вектора АВ, если А(0;3), В(6;6). 2.Напишите уравнение окружности радиуса r с центром А, если А(5;0) и r = 2 3. Найдите координаты вектора n = 2 a – 3 b, где a {2;3}, b {-5;2}. 4. Точка М – середина отрезка АВ. Найдите координаты точки А, если М(-3;-4), В(7;4)

9 класс 2 вариант

1. Лемма о коллинеарных векторах. 2. Координаты суммы векторов. 3. Координаты произведения вектора на число. 4. Координаты середины отрезка. 5. Расстояние между двумя точками 6. Уравнение прямой.

Задачи Найдите координаты вектора с =садиаметром – b, если а 1.1.Напишите уравнение окружности {5;3}, b {3;1} . МN, если М(3;5), N(7;3). Найдите длину вектора АВ,АВС если АВ{6;8}. 2.2.Докажите, что треугольник равнобедренный Найдите координаты m = В(5;4), a + b, если a{7;и3.найдите его периметр, вектора если А(1;1), С(4;-3). 2}, b {-3;1} 3. Докажите, что четырехугольник АВСD – 4. Точка М середина АВ. Найдите прямоугольник и найдитеотрезка его периметр, если координаты точки М, если А(3;2), В(6;7). А(-3;-1), В(1;-1), С(1;-3), D(-3;-3) 5. Найдите координаты вектора АВ, если А(3;2), В(7;5).. 1. Найдите длину вектора АВ, если А(3;2), В(6;8). 2. Напишите уравнение окружности радиуса r с центром А, если А(0;5) и r = 3. 3. Найдите координаты вектора n = 2a – 3b, где a{1;4}, b{-3;5}. 4. Точка М середина отрезка АВ. Найдите координаты точки А, если М(-4;-3), В(4;7).

Зачет № 3 1 вариант

«Скалярное произведение векторов» Вопросы к зачету

1. Основное тригонометрическое тождество. 2. Формулы приведения. 3. Теорема синусов 4. Скалярное произведение векторов в координатах. 5. Косинус угла между векторами 6.Скаляное произведение перпендикулярных векторов

Задачи 1. Вычислите скалярное произведение векторов (а+в) с, если известно, что угол между векторами а 0, а между векторами 0. и и В с равен с равенАВ 450и, 1. ∆ АВС60 АС=10см, LА=750,LC=60в Найдите /a/АВС=. 2, /b/ = 5, /c/ = 6 S 2. Найдите При каком значении х с векторы а А(3;1), и b углы треугольника вершинами перпендикулярны, если а{4;6}, b{x;-6} В(2;-1), С(1;2) 3.Площадь треугольника АВС равна 54см2. Найдите 3. С помощью теоремы косинусов решить 1. треугольник, Найдите площадь треугольника 0 сторону АВ, если LA = 30 . АВС, если еслиАС=18см а=6,0 в=5,ис=9. АВ=6см, АС=4см,теорему LA=30 . синусов, решите АВС, если 4. Используя 2. АВ=10см, Вычислите скалярное 0,LВ=450произведение векторов а LА=30 и b, если / a / = 6, / b / = 4, а угол между ними равен 300. 3. С помощью теоремы синусов найдите а, если в=12,LA = 450, LB = 300. 4. Найдите cosα, если sin α=0,3. 5. Вычислите скалярное произведение векторов a {-4;6}, b {2;3} .

9 класс 2 вариант

1. Формулы приведения. 2.Теорема о площади треугольника. 3. Теорема косинусов. 4. Определение скалярного произведения векторов. 5. Свойства скалярного произведения векторов 6. Косинус угла между векторами.

Задачи 1. Вычислите скалярное произведение векторов (а + в) с, если известно, что угол между а0 и с равен 0 1. треугольнике АС=8см, LA=75 600В , угол между в и сАВС, равен 450, / a / =3,, LC / b =60 / = 7,. Найдите / c / = 4 AB и SABC 2. углы треугольника 2. Найдите При каком значении х с вершинами векторы а А(-3;1), и b B(-2:-1), С(-1;2) перпендикулярны, если а {3;5}, b {x;-15}. 3. С помощью теоремы АВС косинусов решить 3. Площадь треугольника равна 64 см2. треугольник, если а=7, в= 8, с= 9 Найдите сторону АВ, если АС = 16см,АВС, LA = если 300. 1. Найдите площадь треугольника 4. Используя теорему синусов решить треугольник АВ = 3см, АС = 6 см, LA=3000.. АВС, если АВ=15см, LA=30 , LB=450. 2.Вычислите скалярное произведение векторов а, в, если / a / = 5, / b /= 6, а угол между ними равен 300.. 3. С помощью теоремы синусов найдите в, если а=16, LA=300, LB = 450. 4.Найдите cos α , если sin α = 0,2 5. Вычислите скалярное произведение векторов а {3;5}, b {-3;4}

Зачет № 4 1 вариант

«Длина окружности и площадь круга» Вопросы к зачету

1. Определение правильного многоугольника. 2. Определение описанной окружности. 3. Теорема об окружности описанной около правильного многоугольника. 4. Длина окружности. Длина дуги окружности. 5. Формулы для вычисления площади правильного многоугольника, его стороны и радиуса вписанной окружности.

Задачи 1. правильного треугольника, вписанного 1. Периметр Найдите радиус круга, если площадь круга равна в окружность равен 36см. 1. Найдите 10 –Найдите угольника.сторону 28,26 см2. углы правильного квадрата, вокружности ту же окружность. 2. длину дугикольца, радиуса 4см, 2. Найдите Найдитевписанного площадь ограниченного двумя 2. Периметр правильного треугольника, 0 вписанного если ее градусная мера равна 45 . окружностями с общим центром и радиусами 4см и в окружность равен 18см. Найдите углом радиус 3. Найдите площадь сектора с центральным 6см. окружности, 0 вписанной в данный треугольник. 60 3. и радиусом Сколько 6 см. сторон имеет правильный 4. Вычислите площадь круга, его если егоравен радиус многоугольник, если каждый угол 1200. равен 3см. 4. Радиус окружности, описанной около 5. Вычислите длину окружности, если ее радиус правильного четырехугольника равен 8см. Найдите равен 5см. площадь данного четырехугольника. 5. Радиус окружности, описанной около правильного треугольника, равен12см. Найдите сторону треугольника и радиус окружности, вписанной в данный треугольник. 6. Найдите радиус окружности, если длина окружности равна 6,28 см.

9 класс 2 вариант

1. Формула для вычисления угла правильного n – угольника. 2. Определение вписанной окружности. 3.Теорема об окружности вписанной в правильный многоугольник. 4. Площадь круга. Площадь сектора. 5.Формулы для вычисления площади правильного многоугольника, его стороны и радиуса вписанной окружности.

Задачи 1. правильного треугольника, вписанного 1. Периметр Найдите радиус круга, если площадь круга равна в 45 см. 1. окружность Найдите правильного 9 – Найдите угольника.сторону 12,56 см2. углыравен квадрата, вписанного в туокружности же окружность. Найдите длину дуги радиусадвумя 8 см, 2.2.Найдите площадь кольца, ограниченного 2. Периметр правильного треугольника, 0 вписанного если ее градусная мерацентром равна 45и. радиусами 4см и окружностями с общим в3. Найдите окружность равен 27 см. Найдите радиус площадь сектора с центральным углом 5см. окружности, 0 вписанной в данный треугольник. 60 радиусом 3 см. 3. Сколько сторон имеет правильный 0. 4. Вычислите если площадь круга, если его 150 радиус многоугольник, каждый его угол равен равен 2см. 4. Радиус окружности, описанной около 5. Вычислите длину окружности, если ее радиус правильного четырехугольника равен 4 см. равен 2 площадь см. Найдите данного четырехугольника. 5. Радиус окружности, описанной около правильного треугольника, равен 6 см. Найдите сторону треугольника и радиус окружности, вписанной в данный треугольник. 6. Найдите радиус окружности, если длина окружности равна 9,42 см.

Скалярное сложение векторов формула

Сложение векторов. Векторная сумма. Правила сложения векторов. Геометрическая сумма. Он-лайн калькулятор

В механике существуют два типа величин:

  • скалярные величины, задающие некоторое числовое значение — время, температура, масса и т.д.
  • векторные величины, которые вместе с некоторым числовым значением задают направление — скорость, сила и т.д..

Рассмотрим сначала алгебраический подход к сложению векторов.

Покоординатное сложение векторов.

Пусть даны два вектора, заданные покоординатно ( чтобы вычислить координаты вектора, нужно вычесть из соответствующих координат его конца соответствующие координаты его начала, т.е. из первой координаты — первую, из второй — вторую и т.д.):

Тогда координаты вектора, получившегося при сложении этих двух векторов вычисляются по формуле:

В двумерном случае все абсолютно анологично, просто отбрасываем третью координату.

Теперь перейдем к геометрическому смыслу сложения двух векторов: .

При сложении векторов нужно учитывать и их числовые значения, и направления. Есть несколько широко используемых методов сложения:

  • правило параллелограмма
  • правило треугольника
  • тригонометрический способ

Правило параллелограмма.

Процедура сложения векторов по правилу параллелограмма заключается в следующем:

  • нарисовать первый вектор, учитывая его величину и направление
  • от начала первого вектора нарисовать второй вектор, также используя и его величину, и его направление
  • дополнить рисунок до параллелограмма, считая, что два нарисованных вектора — это его стороны
  • результирующим вектором будет диагональ параллелограмма, причем его начало будет совпадать с началом первого (а, значит, и второго) вектора.

Правило треугольника

Сложение векторов по правилу треугольника заключается в следующем:

  • нарисовать первый вектор, используя данные о его длине ( числовой величине) и направлении
  • от конца первого вектора нарисовать второй вектор, также учитывая и его размер, и его направление
  • результирующим вектором будет вектор, начало которого совпадает с началом первого вектора, а конец — с концом второго.

Тригонометрический способ

Результирующий вектор сложения двух компланарных векторов может быть вычислен с помощью теоремы косинусов:

F = числовое значение вектора

α = угол между векторами 1 и 2

Угол между результирующим вектором и одним из исходных векторов может быть вычислен по теореме синусов:

α = угол между исходными векторами

Пример — сложение векторов.

Сила 1 равна 5кН и воздействует на тело в направлении, на 80 o отличающемся от направления действия второй силы, равной 8 кН.

Результирующая сила вычисляется следующим образом:

Fрез = [ (5 кН) 2 + (8 кН) 2 — 2 (5 кН)(8 kН) cos(180 o — (80 o )) ] 1/2

Угол между результирующей силой и первой силой равен:

А угол между второй и результирующей силой можно посчитать следующим образом: as

α = arcsin [ (5 кН) sin(180 o — (80 o )) / (10,2 кН) ]

Он-лайн калькулятор сложения векторов.

Калькулятор ниже может быть использован для любвых векторных величин ( силы, скорости и т.д.) Точка начала вектора совпадает с началами обоих исходных векторов.

Консультации и техническая
поддержка сайта: Zavarka Team

Сложение векторов по правилу треугольника (суммой векторов и называется вектор , начало которого совпадает с началом вектора , а конец — с концом вектора , при условии, что начало вектора приложено к концу вектора ) даёт возможность упрощать выражение перед вычислением произведений векторов.

Сложение векторов, заданных координатами (при сложении одноимённые координаты складываются) даёт возможность узнать, как расположен относительно начала координат вектор, являющийся суммой слагаемых векторов. Подробно эти две операции разбирались на уроке «Векторы и операции над векторами».

Теперь же нам предстоит узнать, как найти длину вектора, являющегося результатом сложения векторов. Для этого потребуется использовать теорему косинусов. Такую задачу приходится решать, например, когда дорога из пункта A в пункт С — не прямая, а отклоняется от прямой, чтобы пройти ещё через какой-то пункт B, а нужно узнать длину предполагаемой прямой дороги. Кстати, геодезия — одна из тех сфер деятельности, где тригонометрические функции применяются во всех их полноте.

При сложении векторов для нахождения длины суммы векторов используется теорема косинусов. Пусть и — векторы, — угол между ними, а — сумма векторов как результат сложения векторов по правилу треугольника. Тогда верно следующее соотношение:

,

где — угол, смежный с углом . У смежных углов одна сторона общая, а другие стороны лежат на одной прямой (см. рисунок выше).

Поэтому для сложения векторов и определения длины суммы векторов нужно извлечь квадратный корень из каждой части равенства, тогда получится формула длины:

.

В случае вычитания векторов () происходит сложение вектора с вектором , противоположным вектору , то есть имеющим ту же длину, но противоположным по направлению. Углы между и и и между и являются смежными углами, у них, как уже было отмечено, одна сторона общая, а другие стороны лежат на одной прямой. В случае вычитания векторов для нахождения длины разности векторов нужно знать следующее свойство косинусов смежных углов:

косинусы смежных углов равны по абсолютной величине (величине по модулю), но имеют противоположные знаки.

Перейдём к примерам.

Пример 1. Векторы и образуют угол . Их длины: и . Выполнить сложение векторов и найти их сумму . Выполнить вычитание векторов и найти их разность .

Решение. Из элементарной тригонометрии известно, что .

Шаг 1. Выполняем сложение векторов. Находим длину суммы векторов, поставляя в формулу длины косинус угла, смежного с углом между векторами:

Шаг 2. Выполняем вычитание векторов. Находим длину разности векторов, подставляя в формулу косинус «изначального» угла:

Выполнить сложение и вычитание векторов самостоятельно, а затем посмотреть решение

Пример 2. Векторы и образуют угол . Их длины: и . Выполнить сложение векторов и найти их сумму . Выполнить вычитание векторов и найти их разность .

Пример 3. Даны длины векторов и длина их суммы . Найти длину их разности .

Шаг 1. По теореме косинусов составляем уравнение, чтобы найти косинус угла, смежного с углом между векторами и находим его:

Не забываем, что косинус смежного угла получился со знаком минус. Это значит, что косинус «изначального» угла будет со знаком плюс.

Шаг 2. Выполняем вычитание векторов. Находим длину разности векторов, подставляя в формулу косинус «изначального» угла:

Пример 4. Даны длины векторов и длина их разности . Найти длину их суммы .

Шаг 1. По теореме косинусов составляем уравнение, чтобы найти косинус «изначального» угла (задача обратная по отношению к примеру 1) и находим его:

Шаг 2. Меняем знак косинуса и получаем косинус смежного угла между и :

Шаг 3. Выполняем сложение векторов. Находим длину суммы векторов, подставляя в формулу косинус смежного угла:

Пример 5. Векторы и взаимно перпендикулярны, а их длины . Найти длину их суммы и и длину их разности .

Два смежных угла, как нетрудно догадаться из приведённого в начале урока определения, в сумме составляют 180 градусов. Следовательно, смежный с прямым углом (90 градусов) угол — тоже прямой (тоже 90 градусов). Косинус такого угла равен нулю, то же самое относится и к косинусу смежного угла. Поэтому, подставляя это значение в выражения под корнем в формуле длины суммы и разности векторов, получаем нули как последние выражения — произведения под знаком корня. То есть длины суммы и разности данных векторов равны, вычисляем их:

Пример 6. Какому условию должны удовлетворять векторы и , чтобы имели место слелующие соотношения:

1) длина суммы векторов равна длине разности векторов, т. е. ,

2) длина суммы векторов больше длины разности векторов, т. е. ,

3) длина суммы векторов меньше длины разности векторов, т. е. ?

Находим условие для первого соотношения. Для этого решаем следующее уравнение:

То есть, для того, чтобы длина суммы векторов была равна длине их разности, необходимы, чтобы косинус угла между ними и косинус смежного ему угла были равны. Это условие выполняется, когда углы образуют прямой угол.

Находим условие для второго соотношения. Решаем уравнение:

Найденное условие выполняется, когда косинус угла между векторами меньше косинуса смежных углов. То есть, чтобы длина суммы векторов была больше длины разности векторов, необходимо, чтобы углы образовали острый угол (пример 1).

Находим условие для третьего соотношения. Решаем уравнение:

Найденное условие выполняется, когда косинус угла между векторами больше косинуса смежных углов. То есть, чтобы длина суммы векторов была меньше длины разности векторов, необходимо, чтобы углы образовали тупой угол.

В статье узнаете что такое вектор, векторные компоненты, единичный вектор, как складывать вектора, умножать вектора на скаляр, скалярное, векторное и смешанное произведение двух векторов.

Сохранение физической величины с вектором обычно означает совершенно иную ситуацию, чем просто сохранение ее скалярной длины. Постоянное значение импульса p (скаляр) может означать совершенно иную ситуацию, чем постоянный вектор p.

Вектор должен иметь три необходимые характеристики: значение (длина), направление, начало и конец.

Любое изменение любого из этих признаков — длины, направления или начало с концом — означает, что создан другой вектор. Два вектора равны тогда и только тогда, когда они имеют равную длину, направление и начало с концом.

Векторные компоненты

Компонентами вектора являются его проекции на оси системы координат.

Также в трехмерном пространстве векторы A называются векторами, которые являются проекциями этого вектора A на оси системы координат.

Имея вектор A, мы погружаем его в систему координат x, y, z. Векторы, являющиеся проекциями вектора A на оси системы, называются векторными компонентами вектора A. Вектор A является векторной суммой составляющих векторов Ax, Ay и Az .

Единичный вектор

Единичный вектор, имеющий то же направление, что и вектор, на который он ссылается, важен, но его длина всегда равна 1.

Единичные векторы осей координат. Мы также присваиваем единичные векторы оси системы отсчета. а) относится к правовращающей системе и б) к левосторонней системе.

Сложение векторов

Сумма вектора обычно не совпадает с суммой скалярных величин:

Добавление двух или более векторов друг к другу сводится к добавлению их компонентов, то есть проекций на опорные оси. Результирующий вектор называется случайным вектором. Для двух векторов результирующий вектор является диагональю параллелограмма, построенного на этих векторах. Метод параллелограмма.

В случае большего числа векторов результирующий вектор получается путем рисования одного из этих векторов, затем в конце первого вектора мы начинаем второй, в конце второго мы даем начало третьего и так далее. Полученный вектор является вектором, начало которого находится в начале первого из добавленных векторов. и его конец в конце последнего. При изменении порядка сложения результирующий вектор (красный) не меняет длину, направление:

Это правило добавления векторов также действует в трехмерном пространстве:

Умножение вектора на скаляр

Самым простым умножением, выполняемым на векторах, является умножение вектора на скаляр (число). Такое умножение не меняет направление вектора, но, как правило, меняет его длину и может изменить его конец (когда скаляр является отрицательным числом). Когда вектор A умножается на α-скаляр, мы получаем новый вектор B:

Скалярное произведение и векторное произведение двух векторов являются очень важными направления в физике и геометрии. Существует также смешанное произведение трех векторов.

Скалярное произведение двух векторов

Формально скалярное произведение векторов представляет собой точку, и ее значение определяется зависимостью

Скалярное произведение описывает способ, которым оба вектора видят друг друга, то есть как долго тень (проекция) отбрасывает каждый из векторов в своего партнера, когда угол между ними равен φ

B cos φ — длина тени, которую вектор B выбрасывает в вектор A. Аналогично, A cos φ — длина тени, которую вектор A выбрасывает в вектор B.

Когда длина проекции (тени) одного из векторов равна нулю, тогда длина проекции второго вектора равна нулю, то есть A • B = 0. Это означает, что эти векторы не работают в одном и том же направлении вообще. Работа, которую мы выполняем при движении автомобиля, зависит не только от приложенной силы F, но и от угла, который создает направление силы и направление пути.

Так как единичные векторы оси системы отсчета х, у и z, которые обозначают векторы ех, еYи еz, перпендикулярны друг к другу, то в виду того, что А • В = АВcosφ и что cos 0 = 1 и cos 90 o = 0, мы получаем произведение значений этих единичных векторов:

Выполнение аналогичного умножения на векторы A и B

мы получили новое выражение для скалярного произведения двух векторов A и B

Значение скалярного произведения двух векторов A и B можно записать в виде двух эквивалентных выражений:

Сравнивая оба выражения, мы находим выражение для угла между векторами A и B:

Векторное произведение двух векторов

Многие важные величины в науке и технике определяются вектором, который является произведением двух других векторов. В таких случаях произведение этих векторов, называемое векторным произведением , приводит к третьему вектору.

В этом случае задача состоит в том, чтобы определить все три особенности вектора C, являющегося произведением векторного произведения векторов A и B:

Произведение векторов A и B , приводящее к третьему вектору C, отмечено диагональным крестом

Направление

Вектор С такой, что вектор перпендикулярен к плоскости, образованной векторами A и B, которая перпендикулярна как к вектору A и B.

Длина

вектор С равен значению параллелограмма, построенного на векторах А и В. Числовой C = ABsin φ.

Начало и конец

Вектор С определяет правое направление движения шнека во время нанесения первого вектора, а именно А или B.

Изменение порядка применения векторов означает изменение знака векторного произведения.

Таким образом, действительное свойство векторного произведения выглядит следующим образом A*B= -B*A

В отличие от скалярного произведения, векторное произведение некоммутативно.

Мы встретимся с векторным произведением на протяжении всего курса физики. Это также часто встречается в механике, а также в науке об электричестве и магнетизме.

В повседневной жизни векторное произведение находится в виде момента силы во вращательном движении. Мы воздействуем на вращательное движение тем эффективнее, чем больше применяем момент силы.

При откручивании гайки гаечным ключом речь идет не только о силе F, но и о способе ее применения (длина рычага R и угол, который создает рычаг с направлением силы).

Все эти зависимости элегантно включены в одно выражение в виде векторного произведения:

Хотя составляющие вектора C, который является произведением векторного произведения векторов A и B, уже включены в его длину и направление, но имея данные составляющих векторов A и B, мы можем использовать их для определения компонентов вектора C в форме матрицы:

Удобнее всего рассчитать этот определитель, расширив относительно первой строки.

Смешанное произведение трех векторов

Смешанное произведение трех векторов является скалярным значением, равным значению детерминанта

Геометрическая интерпретация: смешанное произведение численно равно объему V параллелепипеда, растянутому по векторам A, B и C:

Циклическая корректировка векторов в смешанном произведении не меняет значение этого произведения, то есть:

Тимеркаев Борис — 68-летний доктор физико-математических наук, профессор из России. Он является заведующим кафедрой общей физики в Казанском национальном исследовательском техническом университете имени А. Н. ТУПОЛЕВА — КАИ

Определение вычитание векторов общее значение и понятие. Что это такое вычитание векторов

Вектор — это понятие, которое имеет несколько применений. Это может быть агент, который отвечает за перемещение объекта с одного сайта на другой; проекции с интенсивностью и характеристиками, которые варьируются; величины, которая имеет точку приложения, смысл и адрес; или организма, способного передавать определенные заболевания.

То есть вектор — это инструмент, который дает возможность осуществлять представление векторных величин, которые нуждаются не только в смысле, но и в направлении, а также в определенной величине.

Понятие вычитания векторов используется в математике . В этом случае вектор представляет собой величину, которая изображается как сегмент, который имеет свое начало в точке A и ориентирован к ее концу ( точка B ). Следовательно, вектор является отрезком AB .

Вычитание векторов — это операция, которая выполняется с двумя из этих сегментов . Чтобы выполнить вычитание двух векторов, нужно взять ректора и добавить его противоположность .

Предположим, мы хотим выполнить следующее вычитание: AB — DE, то есть AB (-3, 4) и DE (5, -2) в соответствии с положением векторов в декартовой плоскости . Учитывая сказанное о сумме обратного, мы должны рассмотреть операцию следующим образом:

(-3, 4) — (5, -2)
(-3-5, 4 + 2)
(-8, 6)

Как вы можете видеть, в -3 мы добавляем противоположность 5 (то есть -5 ), а в 4 мы добавляем противоположность -2 (то есть 2 ). Таким образом, результатом этого вычитания векторов является (-8, 6) .

Если, с другой стороны, мы добавили векторы, операция была бы проще, так как было достаточно добавить компоненты:

(-3, 4) + (5, -2)
(-3 + 5, 4-2)
(2, 2)

Считается, что сложение векторов гораздо проще, чем вычитать их. И что для того, чтобы выполнить первую упомянутую операцию, единственное, что нужно сделать, — это запустить вторую после того, что является концом первого, началом третьего с того, что является концом второй и так последовательно, пока не будут использованы все векторы, с которыми вы хотите работать.

Другими важными аспектами, которые следует учитывать относительно векторов и операций, которые можно выполнять с ними, являются следующие:
-Сумары, вычитания и умножения являются операциями, которые могут быть выполнены с ними.
— При переходе к сложению или вычитанию векторов достигается получение другого вектора, и это может быть достигнуто с помощью различных типов процедур, числовых или геометрических.
Вычитание может быть выполнено через заданные декартовы координаты векторов, как в пространстве, так и в том, что будет плоскостью.
-Сложение и вычитание векторов в пространстве могут быть объединены.
Противоположность любого вектора всегда имеет ту же меру, что и этот, но в противоположном направлении.

Сложение векторов — свойства, правила и примеры решения задач

Отрезок, который имеет направление, называется вектором. По сути, эта линия, характеризующаяся определённой длиной. Так как с математической точки зрения это выражение, то с ним можно выполнять различные операции. Простейшими являются действия вычитания двух и более векторов и их сложение. Выполняются они по правилам геометрии и алгебры.

Общие сведения

Понятие вектор используется как в физике, так и в математике. С его помощью обозначают действие различных сил, указывают их направление, определяют движение. По сути, это величина, противопоставляемая массе, объёму, плотности, температуре, то есть «скалярам». Согласно определению вектор — это отрезок, имеющий строгое направление. Точку, из которой он выходит, называют начальной, а в которой заканчивается — конечной.

Обозначают отрезок помощью заглавных латинских букв, сверху которых ставится чёрточка. Рисуют же его с помощью прямой ограниченной линии.

Например, запись AB обозначает, что точка A является началом, а B концом. В некоторых случаях для кратности отрезки допустимо обозначать одной маленькой буквой, так: AB = a.

Векторная запись используется тогда, когда невозможно величины описать с помощью одного числа. Численное значение выражение определяется длиной отрезка или его модулем. Эта величина является скалярной. В том случае если начало и конец ограниченной линии совпадают, то говорят о нулевой линии. Обозначают её цифрой 0.

Векторы, расположенные на плоскости или в пространстве, по отношению друг к другу могут быть:

  • коллинеарными — отрезки лежат на одной линии или ей параллельны;
  • соноправленными — замкнутые линии направление которых одинаковое;
  • противоположными — вектора направлены в разные стороны;
  • ортогональными — перпендикулярными друг другу;
  • компланарными — лежащими на одной плоскости или ей параллельные;
  • равными — ограниченными прямыми, совпадающими как по направлению, так и по величине.

Так как вектора — это выражения, то с ними можно выполнять различные действия. Их возможно складывать, вычитать, умножать на число. При работе с векторными величинами используют декартовую систему координат. В ней прямую замкнутую линию раскладывают по базису и определяют координаты её точек. Другими словами, выполняют проекции отрезков на оси. Непосредственно за базис берут орты.

Если известны начальные координаты и конечные, то текущие вычисляют путём вычитания из последних первые. Существующая возможность записать любое геометрическое свойство, используя координаты, позволяет отойти от геометрии и использовать для вычислений алгебру.

Сложение координат

Существует простое правило применимое для направленных отрезков и позволяющее найти их сумму. Заключается оно в следующем: если необходимо прибавить один вектор к другому описывающийся каждый своими координатами, достаточно сложить соответствующие их орты. Например, предположим есть два вектора a и b. Первый отрезок имеет координаты (ax; ay), а второй (bx;by). При их сложении получится новый вектор c. В результате действия его координаты будут c (ax + bx; ay + by).

Это теорема доказывается просто. Пусть даны отрезки f (x 1; y 1) и g (x 2; y 2). В системе координат относительно рассматриваемых векторов получится: f = x 1 a + y 1 b; g = x 2 a + y 2 b. Тогда искомая сумма будет: f + g = x1a + y1b + x2a + y2b = a (x 1 + x 2) + b (y 1 + y 2). Что и нужно было доказать. Это правило применимо к векторам имеющим любые координаты. Например, пусть есть a (1; 2), b (-3; 1). Нужно найти их сумму. С помощью формулы сложения получится новый направленный отрезок с координатами a + b = (1 — 3; 2 + 1) = (-2; 3).

Как и при операциях с простыми числами при работе с векторными выражениями используют различные их свойства. Существует три правила сложения векторов:

  • При выполнении операции очерёдность слагаемых значения не имеет: a + b = b + a. То есть от перемены мест слагаемых результат не изменится.
  • Если необходимо к сумме векторов прибавить третий, то сложение можно выполнить в любой очерёдности: (a + b) + c = a + (b + c) = (a + c) + b.
  • При сложении векторного выражения с вектором, не имеющим длины и направления, исходные координаты не изменятся: a + 0 = a, где 0 (0; 0).
  • Приведённые свойства соответственно называют переместительным, сочетательным, нулевым законом. Например, предположим есть два направленных отрезка a (2; 2) и b (-4; 1). Согласно первому свойству, очерёдность значения не имеет, поэтому что при прибавлении b к a, что при a к b результат будет одинаковый: a + b = (2 -4; 2 + 1) = (-2; 3), b + a = (-4 + 2; 1 +2) = (-2; 3). По аналогии можно проверить правильность утверждения и двух оставшихся свойств.

    Следует отметить, что при сложении двух противоположных ограниченных прямых сумма будет равняться нуль-вектору: a + (-a) = 0. Это утверждение не требует доказательства, так как здесь используется фундаментальный закон алгебры — правило знаков.

    Правило параллелограмма

    По сути, все операции с векторными выражениями сводятся к их приращению или уменьшению. Если координаты точек неизвестны, то алгебраический метод складывания не подходит. В таком случае используют геометрические операции. Одним из способов, позволяющих сложить два неколлинеарных вектора, является правило параллелограмма или прямоугольника при перпендикулярном направлении складываемых отрезков.

    Сформулировать способ можно следующим образом: если имеются два отрезка не лежащие на параллельной прямой и не принадлежащие ей, то нужно достроить данные вектора до параллелограмма. Для этого необходимо взять произвольную точку и отложить от неё отрезок AB равный первому вектору, и AD совпадающий со вторым. При этом необходимо придерживаться соотношения геометрии наклона. Затем достроить необходимые параллельные прямые таким образом, чтобы образовался параллелограмм ABCD. Если в такой фигуре провести диагональ, то её длина и будет равняться сумме складываемых отрезков.

    Доказать правильность утверждения можно следующими доводами. Пусть имеются две ограниченные линии a и b. От точки A можно отложить первый отрезок конец, которого обозначить как B, и второй, с точкой D. Теперь через D и B возможно провести соответственно параллельные прямые AB и AD. Место, в которой они пересекутся, пусть будет обозначено как С. Тогда используя признак параллельности двух пар прямых в фигуре ABCD, можно утверждать, что это параллелограмм. Вектор AC = a + b. Это следует из равенства отрезков AD = BC и теоремы о подобных треугольниках.

    Пример задания. Определить, чему равна сумма двух отрезков длиной 2 см и 1 см расположенные друг к другу под углом 45. Для того чтобы воспользоваться правилом, нужно взять листочек в клеточку и построить два вектора, исходящие из одной точки O. Тогда первый отрезок будет OA, а второй OB. Затем достроить прямые таким образом, чтобы на рисунке получился параллелограмм. Новая полученная точка пусть будет D. Теперь с помощью линейки можно измерить диагональ фигуры, длина которой и будет искомой суммой. В ответе должно получиться, что OA + OB = OD = 3 см.

    Простыми словами это правило можно рассказать так: сумма двух отрезков будет равняться диагонали параллелограмма, построенного на исходных векторах. Эта теорема чаще используется не в геометрии, а физике, например, при сложении сил.

    Альтернативные методы

    Операцию по сложению двух векторов можно выполнить и с помощью правила треугольника. Делается это так. Выбирается любая точка на плоскости, от которой откладываются два вектора. При этом необходимо соблюдать их размерность и наклон по отношению друг к другу. Затем две конечные точки соединяют прямой. Её длина и будет искомой величиной. То есть в итоге должна получиться равнобедренная фигура.

    Применение метода сложения векторов по правилу треугольника позволяет довольно легко находить сумму для трёх и более отрезков. Для этого сначала вычисляют результат сложения для двух любых линий, а после прибавляют к полученной ограниченной прямой третью и так далее.

    При сложении нескольких векторов удобно выполнять следующую последовательность построений:

    • от выбранной точки пространства рисуется вектор, равняющийся первому слагаемому;
    • от конечной точки откладывается вектор, совпадающий со вторым слагаемым;
    • приведённая последовательность потеряется необходимое число раз;
    • прямой линией соединяется точка, с которой началось построение с конечной последнего вектора;
    • длина полученного отрезка и будет являться результатом сложения.

    Этот способ получил название метод многоугольника. Он довольно часто применяется на практике, позволяя, довольно просто выполнить нахождение суммы. Из правила треугольника, а, следовательно, и многоугольника, вытекает следствие, которое подтверждает, что если складывается отрезок с нулевым векторным выражением, то в ответе получится длина, совпадающая со значимым слагаемым.

    Следует отметить, что методы используются только, если направление отрезков является сонаправленным.

    Если же отрезки неколлинеарные, то от конца одного откладывается другой. Тогда искомая сумма будет равняться длине линии, первой точкой которой будет начало одной векторной прямой, а конец совпадать с точкой, завершающей другую. То есть сумма — это отрезок, начало которого совпадает с началом обеих линий, а длина равна разности их длин, при этом направление его будет совпадать с тем что больше по длине.

    Предыдущая

    ГеометрияПлощадь треугольника по координатам вершин — формулы для расчета

    Следующая

    ГеометрияПлощадь прямоугольной трапеции через угол и основания

    Зачеты по геометрии 9 класс презентация, доклад

    Текст слайда:

    1 вариант Вопросы к зачету 2 вариант

    1. Основное тригонометрическое тождество.
    2. Формулы приведения.
    3. Теорема синусов
    4. Скалярное произведение векторов в координатах.
    5. Косинус угла между векторами
    6.Скаляное произведение перпендикулярных векторов

    Зачет № 3 «Скалярное произведение векторов» 9 класс

    Задачи Задачи

    1. Найдите площадь треугольника АВС, если АВ=6см, АС=4см, LA=300.
    2. Вычислите скалярное произведение векторов а и b, если / a / = 6, / b / = 4, а угол между ними равен 300.
    3. С помощью теоремы синусов найдите а, если в=12,LA = 450, LB = 300.
    4. Найдите cosα, если sin α=0,3.
    5. Вычислите скалярное произведение векторов
    a {-4;6}, b {2;3} .

    1. Вычислите скалярное произведение векторов (а+в) с, если известно, что угол между векторами а и с равен 600, а между векторами в и с равен 450, /a/ = 2, /b/ = 5, /c/ = 6
    2. При каком значении х векторы а и b перпендикулярны, если а{4;6}, b{x;-6}
    3.Площадь треугольника АВС равна 54см2. Найдите сторону АВ, если АС=18см и LA = 300.

    1. В ∆ АВС АС=10см, LА=750,LC=600. Найдите АВ и SАВС.
    2. Найдите углы треугольника с вершинами А(3;1), В(2;-1), С(1;2)
    3. С помощью теоремы косинусов решить треугольник, если а=6, в=5, с=9.
    4. Используя теорему синусов, решите АВС, если АВ=10см, LА=300,LВ=450

    1. Формулы приведения.
    2.Теорема о площади треугольника.
    3. Теорема косинусов.
    4. Определение скалярного произведения векторов.
    5. Свойства скалярного произведения векторов
    6. Косинус угла между векторами.

    Найдите площадь треугольника АВС, если
    АВ = 3см, АС = 6 см, LA=300..
    2.Вычислите скалярное произведение векторов а, в, если / a / = 5, / b /= 6, а угол между ними равен 300..
    3. С помощью теоремы синусов найдите в, если а=16, LA=300, LB = 450.
    4.Найдите cos α , если sin α = 0,2
    5. Вычислите скалярное произведение векторов а {3;5}, b {-3;4}

    1. Вычислите скалярное произведение векторов (а + в) с, если известно, что угол между а и с равен 600, угол между в и с равен 450, / a / =3, / b / = 7, / c / = 4
    2. При каком значении х векторы а и b перпендикулярны, если а {3;5}, b {x;-15}.
    3. Площадь треугольника АВС равна 64 см2. Найдите сторону АВ, если АС = 16см, LA = 300.

    1. В треугольнике АВС, АС=8см, LA=750, LC =600. Найдите AB и SABC
    2. Найдите углы треугольника с вершинами А(-3;1), B(-2:-1), С(-1;2)
    3. С помощью теоремы косинусов решить треугольник, если а=7, в= 8, с= 9
    4. Используя теорему синусов решить треугольник АВС, если АВ=15см, LA=300, LB=450.

    Сложение векторов — объяснение и примеры

    Подобно скалярному сложению, сложение векторов включает объединение двух или более векторов. Более конкретно, когда вы складываете векторы, вы:

    «Добавление двух или более векторов с помощью операции сложения для получения нового вектора, равного сумме двух или более векторов».

    В этом разделе мы обсудим сложение векторов со следующих аспектов:

    • Что такое сложение векторов?
    • Как добавить векторы графически
    • Как добавить два вектора

    Что такое сложение векторов?

    Два вектора, A и B , могут быть сложены вместе с помощью сложения векторов, и результирующий вектор может быть записан как:

    R = A + B

    Как добавлять векторы графически

    При использовании сложения векторов мы должны учитывать оба компонента вектора, а именно направление и величину.

    Имейте в виду, что два вектора с одинаковой величиной и направлением можно складывать как скаляры.

    В этом разделе мы рассмотрим графические и математические методы сложения векторов, в том числе:

    1. Сложение векторов с использованием правила «голова к хвосту»
    2. Сложение векторов с использованием метода параллелограмма
    3. Сложение векторов с использованием компонентов

    Сложение векторов с использованием правила «голова к хвосту»

    Сложение векторов может быть выполнено с использованием известного метода «голова к хвосту».Согласно этому правилу, два вектора можно сложить вместе, поместив их вместе так, чтобы голова первого вектора соединялась с хвостом второго вектора. Результирующий вектор суммы затем может быть получен путем соединения хвоста первого вектора с заголовком второго вектора. Иногда это также называют методом сложения векторов треугольником.

    Сложение векторов с использованием правила «голова к хвосту» показано на рисунке ниже. Два вектора P и Q складываются с использованием метода «голова к хвосту», и мы можем видеть треугольник, образованный двумя исходными векторами и вектором суммы.

    Сначала два вектора P и Q помещаются вместе так, что голова вектора P соединяется с хвостом вектора Q . Затем, чтобы найти сумму, рисуется результирующий вектор R , который соединяет хвост P с головой Q .

    Математически сумма или результирующий вектор, R, на изображении ниже может быть выражена как:

    R = P + Q

    Сложение векторов с использованием метода параллелограмма

    Чтобы понять сложения векторов методом параллелограмма мы рассмотрим и поясним рисунок ниже.

    Сначала нарисуйте данные векторы, A, и B, , чтобы они имели ту же начальную точку, как показано на изображении ниже. Затем нарисуйте параллелограмм, используя копии данных векторов.

    Во-вторых, нарисуйте копию вектора B под названием B ’, и поместите ее параллельно вектору B, чтобы соединить с головой первого вектора, A . Точно так же нарисуйте копию вектора A под названием A ’, и поместите ее параллельно A так, чтобы его хвост соединялся с головой вектора B.

    Наконец, результат двух векторов, равный сумме векторов A и B , будет диагональю параллелограмма. Его можно нарисовать, соединив начальную точку двух векторов A и B с головкой векторов A ’ и B’ .

    Таким образом, для выполнения сложения векторов с использованием метода параллелограмма требуется три шага:

    Шаг 1: Поместите два вектора так, чтобы они имели общую начальную точку

    Шаг 2: Нарисуйте и завершите параллелограмм, используя копии двух исходных векторов

    Шаг 3: Тогда диагональ параллелограмма равна сумме двух векторов

    Сложение векторов с использованием компонентов

    Как мы знаем, векторы, заданные в декартовых координатах, можно разложить на их горизонтальные и вертикальные компоненты.Например, вектор P под углом Φ, как показано на изображении ниже, можно разложить на его компоненты следующим образом:

    P x , который представляет компонент вектора P вдоль горизонтальной оси. (ось x) и

    P y , который представляет компонент вектора P вдоль вертикальной оси (ось y).

    Можно видеть, что три вектора образуют прямоугольный треугольник и что вектор P может быть выражен как:

    P = P x + P y

    Математически , компоненты вектора также могут быть вычислены с использованием величины и угла данного вектора.2

    И угол может быть найден как:

    Φ = tan -1 (P y / P x )

    Таким образом, в итоге мы можем определить результирующий вектор, если его компоненты заданы. В качестве альтернативы, если задан сам вектор, мы можем определить компоненты, используя приведенные выше уравнения.

    Точно так же, если векторы выражены в упорядоченных парах (векторах-столбцах), мы можем выполнить операцию сложения с векторами, используя их компоненты. Например, рассмотрим два вектора M и N , заданных как:

    M = (m1, m2)

    N = (n1, n2)

    Выполнение векторного сложения двух векторов эквивалентно для добавления соответствующих компонентов x и y двух векторов.2

    Угол можно вычислить как:

    Φ = тангенс угла -1 (S y / S x ).

    Как сложить два вектора

    В этом разделе обсуждаются примеры сложения векторов и их пошаговые решения, чтобы получить некоторую практику использования различных методов, описанных выше.

    Примеры

    Пример 1

    Учитывая, что два вектора, A и B, как показано на изображении ниже, графически определяют свою сумму с использованием метода «голова к хвосту».

    Решение

    Первый шаг метода «голова к хвосту» — это размещение данных векторов A и B таким образом, чтобы хвост вектора B соединялся с головой вектора . A, , как показано на изображении ниже. Затем, чтобы найти их сумму, мы рисуем результирующий вектор R так, чтобы он соединял хвост вектора A с головой вектора B . Математически результат может быть выражен как:

    R = A + B

    Пример 2

    Учитывая два вектора, AB = (3, 2) и BC = ( 2, 2), определите их сумму по правилу «голова к хвосту».

    Решение

    AB + BC = (3, 2) + (2, 2)

    AB + BC = (3 + 2, 2 + 2)

    AB + BC = (5, 4).

    Или, как показано на изображении ниже, результирующий вектор может быть записан как:

    AC = (5, 4)

    Примечание. Чтобы использовать правило треугольника / правило «голова к хвосту», промежуточная буква два добавляемых вектора должны быть одинаковыми:

    AC = AB + BC

    В этом примере промежуточная буква B .2

    | AC | = √ 25 + 16

    | AC | = 6,403 единиц (приблизительно).

    Угол результирующего вектора AC можно найти следующим образом:

    Φ = tan -1 (AC y / AC x )

    Φ = tan -1 (4/5 )

    Φ = tan -1 (4/5)

    Φ = 38,66 градусов

    Пример 3

    Для двух векторов S = 10 м, Φ = 30 градусов и T = 20 м , Φ = 60 градусов, определить их сумму.Затем вычислите величину и угол результирующего вектора, используя метод компонентов.

    Решение

    Пусть R будет результирующим вектором, равным сумме данных векторов, которая может быть выражена как:

    R = S + T

    Для использования компонентного метода , мы сначала смотрим на составные части данных векторов. Горизонтальный компонент S равен:

    Sx = S Cos Φ

    Sx = 10 Cos 30

    Sx = 8.660 м (приблизительно)

    Аналогично для вертикальной составляющей:

    Sy = S Sin Φ

    Sy = 10 Sin 30

    Sy = 5 м

    Далее вычисляем компоненты вектора T:

    Tx = T Cos Φ

    Ty = T Sin Φ

    Где,

    Tx = 20 Cos 60

    Tx = 10m

    60

    Ty = 17.320 (приблизительно)

    Теперь мы можем вычислить вектор суммы, сложив отдельные компоненты x и y вектора S и T следующим образом:

    Rx = Sx + Tx

    Rx = 8,660 + 10

    Rx = 16,660 м

    Ry = Sy + Ty

    Ry = 5 + 17,32

    Ry Результат = 22,3 R можно представить в виде столбца как:

    R = (16.2

    | R | = 23,292 м (приблизительно)

    Φ = tan -1 (Ry / Rx)

    Φ = tan -1 (22,32 / 16,66)

    Φ = 53,26 градуса (приблизительно)

    Таким образом, результат вектор суммы:

    R = 23,292 м, Φ = 53,26 градуса.

    Пример 4

    Путешественник идет P = 20 м прямо на запад, а затем Q = 10 м прямо на север. Определите, как далеко путешественник находится от отправной точки.Также укажите величину и угол результирующего вектора.

    Решение

    Сначала мы графически представляем заданные векторы смещения P и Q, а затем рисуем их результирующий вектор, используя правило «голова к хвосту», как показано на изображении ниже. Из изображения видно, что путешественник преодолел расстояние, равное величине вектора R от начальной точки.

    Теперь, чтобы математически вычислить результирующий вектор, мы используем следующие формулы:

    R = P + Q

    | R | = √ (20) ^ 2 + (10) ^ 2

    | R | = 22.36 м (приблизительно)

    Угол можно рассчитать как:

    Φ = тангенциальный угол -1 (10/20)

    Φ = 26,57 градуса

    Таким образом, путешественник преодолел расстояние 22,36 м с момента старта. точка под углом 26,57 градуса к северо-западу.

    Пример 5

    Определите результирующий вектор суммы для двух векторов A = (-5, -1) и B = (2, -1).

    Решение

    Указанные векторы уже находятся в своей компонентной форме, поэтому сначала мы определяем их углы.

    Для вектора A:

    Φ = tan -1 (Ay / Ax)

    Φ = tan -1 (-1 / -5)

    Φ = 11,31 градуса.

    Для вектора B:

    Φ = tan -1 (By / Bx)

    Φ = tan -1 (-1/2)

    Φ = -26,57 градусов.

    Затем мы находим результирующий вектор, складывая отдельные компоненты:

    S = A + B

    Sx = Ax + Bx

    Sx = -5 + 2

    Sx = -3

    Sy = Ay + By

    Sy = -1 -1

    Sy = -2

    Результирующий вектор S может быть выражен в виде столбца вектор:

    S = (-3, -2).2

    | S | = 3,605 единиц (приблизительно)

    Φ = tan -1 (Sy / Sx)

    Φ = tan -1 (-2 / -3)

    Φ = 33,69 градусов

    Таким образом, итоговая сумма вектор:

    S = 3,605 единиц, Φ = 33,69 градуса.

    Пример 6

    Учитывая два вектора PQ и QR, , как показано на изображении ниже, вычислите их суммарное значение, вектор PR .

    Решение

    Из данного изображения результирующий вектор может быть представлен как:

    PQ + QR = (2, 3) + (2, -2)

    PQ + QR = (4, 1).2

    | PR | = √ 17

    | PR | = 4,123 единицы (приблизительно)

    Угол результирующего вектора PR можно найти следующим образом:

    Φ = тангенциальный угол -1 (1/4)

    Φ = 14,04 градуса

    Практические вопросы
    1. Учитывая два вектора, V = (2, 5) и C = (3, -2), определите их сумму, используя правило «голова к хвосту». Также определите величину и угол результирующего вектора R .
    2. Учитывая два вектора G = (5, 5) и H = (4, -10), определите их сумму, используя правило «голова-йо-хвост». Также определите величину и угол результирующего вектора P .
    3. Учитывая векторы OA, , где O = (-1, 3) и A = (5,2), и вектор UV, , где U = (1, -2) и V = (-2,2), определите результирующий вектор суммы S. Затем найдите его величину и угол.
    4. Учитывая четырехугольник ABCD, определите следующее:
    • DC + CA =?
    • BD + DC =?
    • AD + DC =?

    1. M = 10 м восточной долготы и северной широты = 15 м северной широты.Определите сумму двух векторов, затем найдите величину и угол результирующего вектора.

    Ответы

    1. Результирующий вектор R равен R = (5, 3), величина R равна | R | = 5,830 единиц, а угол Φ = 30,96 градуса.
    2. Результирующий вектор P равен P = (9, 5), величина P равна | P | = 10. 30 единиц, а угол Φ = 29.05 градусов.
    3. Векторы: OA = (6, -1) и UV = (-3, 4), результирующий вектор суммы S задается как S = (3, 3), величина S есть | S | = 4,242 единицы, а угол Φ = 45 градусов.
    4. В данном четырехугольнике сумма вычисляется как:

    DC + CA = DA

    BD + DC = BC

    AD DC + AC

    1. Результат двух векторов:

    R = M + N

    | R | = 18.027 м,

    Угол может быть вычислен как:

    Φ = тангенциальный угол -1 (15/10)

    Φ = 56,30 градуса.

    Таким образом, результирующий вектор равен R = 18,027 м , Φ = 56,30 градусов к северо-востоку.

    Предыдущий урок | Главная страница | Следующий урок

    Как сложить алгебраически два вектора A и B? — Mvorganizing.org

    Как алгебраически сложить два вектора A и B?

    Чтобы сложить или вычесть два вектора, сложите или вычтите соответствующие компоненты.Пусть → u = ⟨u1, u2⟩ и → v = ⟨v1, v2⟩ — два вектора. Сумма двух или более векторов называется результирующей. Результат двух векторов может быть найден с помощью метода параллелограмма или метода треугольника.

    Как сложить или вычесть полярную форму?

    Чтобы сложить комплексные числа в прямоугольной форме, сложите действительные компоненты и сложите мнимые компоненты. Вычитание аналогично. Чтобы умножить комплексные числа в полярной форме, умножьте величины и сложите углы. Чтобы разделить, разделите величины и вычтите один угол из другого.

    Как найти сумму двух векторов?

    Результат находится путем сложения векторов. Вы можете использовать триггерные функции, чтобы найти компоненты x и y каждого вектора, сложить их и найти величину нового вектора… или вы можете заметить, что 110 ° — 20 ° = 90 °. Итак, у вас есть прямоугольный треугольник 3, 4, 5. Величина 5.

    .

    Как складывать векторы под разными углами?

    Чтобы сложить два вектора, сложите их в виде координат: (3.5, 3.5) + (5.7, 4,0) = (9,2, 7,5). Преобразуйте (9.2, 7.5) в форму величины / угла. Примените уравнение theta = tan – 1 (y / x), чтобы найти угол, который равен tan – 1 (7,5 / 9,2) = tan – 1 (0,82) = 39 градусов.

    Каково направление вектора нормали?

    Вектор нормали, часто называемый просто «нормалью», к поверхности — это вектор, перпендикулярный поверхности в данной точке. Когда нормали рассматриваются на закрытых поверхностях, обычно различают направленную внутрь нормаль (указывающую внутрь поверхности) и направленную наружу нормаль.

    Какой вектор между двумя точками?

    Найдите вектор направления по двум точкам: пример вопроса № 3. Найдите вектор, который имеет начальную и конечную точки. Объяснение: Чтобы найти вектор направления, вычтите координаты начальной точки из координат конечной точки.

    Как узнать, параллельны ли векторы?

    Чтобы определить, являются ли они параллельными или они, мы можем проверить, могут ли их соответствующие компоненты быть выражены как скалярные кратные друг другу или нет.Поскольку вектор P в 2 раза больше вектора Q, два вектора параллельны друг другу, а направление вектора Q противоположно направлению вектора P.

    Что означает параллельность двух векторов?

    Два вектора u и v называются параллельными, если они имеют одинаковое или противоположное направление. Это означает, что каждый является скалярным, кратным другому: для некоторого ненулевого скаляра s, v = su, и поэтому u = v.

    Как вы показываете, что векторы коллинеарны?

    Доказать, что векторы a, b и c коллинеарны, если и только если векторы (a-b) и (a-c) параллельны.В противном случае, чтобы доказать коллинеарность векторов, мы должны доказать (a-b) = k (a-c), где k — постоянная.

    Коллинеарность означает параллельность?

    Параллельные векторы — это векторы, которые имеют одинаковую или параллельную опору. Они могут иметь равные или неравные величины, а их направления могут быть одинаковыми или противоположными. Два вектора коллинеарны, если они имеют одинаковое направление, параллельны или антипараллельны.

    Сложение и вычитание векторов

    При сложении двух (или более) векторов всегда приводит к другому вектору, который называется результирующим .Суммируемые векторы известны как компонентов результирующего вектора. Однако, если векторы-компоненты не действуют в одном и том же направлении с самого начала, направление результирующего будет отличаться от направления любого из его компонентов. Кроме того, абсолютная величина результирующего (то есть его величина без учета знака) обычно будет меньше, чем сумма абсолютных величин его составляющих векторов (если только составляющие вектора , все действуют в одном направлении).Вы, вероятно, уже знаете, что любой вектор в двумерной плоскости может быть описан в терминах его компонентов x и y , и что эти компоненты сами являются векторами. Чтобы представить себе, что это означает, представьте, что вы идете три километра на восток, затем четыре километра на север. Всего вы прошли семь километров, но как далеко вы от того места, откуда начали, по прямой?


    Как далеко вы от точки происхождения?


    Глядя на диаграмму выше, ответ (пять километров) можно найти достаточно легко, применив теорему Пифагора.Каждый этап вашего путешествия (три километра на восток и четыре километра на север) представляет собой отдельный вектор . Два вектора образуют катеты прямоугольного треугольника, для которого отрезок прямой, соединяющий точку, в которой вы начали, и точку, в которой вы закончили (т.е. результат ) становится гипотенузой. Вот расчет:

    расстояние от начала координат = √3 2 + 4 2 = 5

    Итак, хотя вы на самом деле прошли семь километров, на самом деле вы находитесь всего в пяти километрах от пункта отправления.Это, конечно, совершенно логично, поскольку кратчайшее расстояние между двумя точками всегда является прямой линией. Мы можем использовать этот вид визуального представления, чтобы найти результат для сложения любого набора векторов. На рисунке ниже показаны два свободных вектора: a и b .


    Бесплатные векторы a и b


    Если мы хотим визуализировать, что происходит, когда мы складываем эти векторы вместе, мы можем переместить один из векторов (не меняя его направления) так, чтобы его хвост касался наконечника стрелки, принадлежащей другому вектору.На самом деле не имеет значения, какой вектор мы выберем для перемещения — результат будет таким же (как и для сложения скалярных значений, сложение векторов равно коммутативному и ассоциативному ). Чтобы показать, что это так, на иллюстрации ниже показаны оба возможных сценария. В каждом случае результат показан зеленой стрелкой.


    Графическое изображение сложения векторов


    Мы можем сделать что-то очень похожее, чтобы представить , вычитая одного вектора из другого.Если мы хотим вычесть вектор b из вектора a , например, мы просто меняем направление вектора b , чтобы создать новый вектор с именем -b , и следуем той же процедуре, что и раньше, то есть перемещаем вектор -b так, чтобы его хвост касался наконечника стрелки, принадлежащей вектору a . Однако обратите внимание, что, как и для скалярных значений, вычитание не является ни коммутативным, ни ассоциативным. Мы не получим того же результата, если вместо этого вычтем вектор a из вектора b .Чтобы продемонстрировать это, ниже проиллюстрированы оба сценария. Обратите внимание, что хотя величина результирующей величины одинакова в обоих случаях, его направление , когда вектор a вычитается из вектора b , полностью противоположно направлению вектора, который получается в результате вычитания вектора b из вектора в .


    Вычитание векторов некоммутативно и неассоциативно.


    Здесь следует отметить, что если мы хотим вычесть один вектор из другого, мы можем просто изменить направление вычитаемого вектора, а затем сложить этих двух векторов вместе.Также следует отметить, что результат идет от выбранной исходной точки до конечной точки непосредственно . Компонентные векторы (то есть векторы, которые мы складываем вместе) создают косвенный путь от одной и той же исходной точки до той же конечной точки. Следовательно, логически сумма компонентов x двух добавляемых векторов должна равняться компоненту x результирующего, а сумма компонентов y векторов должна равняться компоненту y вектора результирующий.Это означает, что мы можем выразить сложение векторов с помощью чисел, вместо того, чтобы рисовать диаграмму каждый раз, когда мы хотим сложить векторы вместе. Давайте снова выполним наше исходное векторное сложение, на этот раз используя матричное сложение, чтобы найти компоненты x и y результирующего:

    Если мы хотим вычесть вектор b из вектора a , мы просто инвертируем значения компонентов x и y для вектора b , как показано здесь:

    Точно так же мы можем вычесть вектор a из вектора b , инвертируя значения компонентов x и y для вектора a :

    Если вы сверите полученные здесь результаты с иллюстрациями выше, вы увидите, что они согласуются с результатами, полученными нами с использованием визуального представления векторного сложения.Пока вы это делаете, вы также можете убедиться, что компоненты x и y , указанные для векторов a и b , верны (вы можете сделать это, подсчитав количество квадратов в каждом из горизонтальных и вертикальные направления между концом стрелки, представляющей вектор, и его кончиком). Для удобства мы использовали целые значения для компонентов x и y всех векторов. В реальной жизни значение компонента часто имеет дробную часть, но принцип тот же.Сложить векторы относительно просто, если вы знаете значения компонентов x и y каждого вектора.

    Хотя до сих пор мы сложили вместе только два вектора, этот процесс одинаково хорошо работает для любого количества векторов. Рассмотрим сценарий, проиллюстрированный ниже, который показывает четыре свободных вектора. Как вы думаете, что будет в результате их сложения?


    Что будет в результате добавления этих векторов?


    Конечно, мы могли бы найти ответ, используя визуальный метод, который мы использовали ранее, но давайте сначала попробуем сложение векторов, используя численный метод.Нам нужно знать компоненты x и y для каждого вектора, которые мы можем найти путем подсчета квадратов. Вот список векторов с их компонентами x и y в виде вектор-строки:

    a = (-4, 2)

    b = (2, 1)

    c = (5, -1)

    d = (-3, -2)

    Теперь у нас есть необходимая информация, и мы можем представить все в виде матрицы:

    a + b + c + d =-4 + 2 + 9123 912 = 0
    2 1 -1-2 0

    Согласно вышеприведенному расчету, сложение векторов a , b , c и d дает результат, для которого оба компонента x и y равны нулю.Единственный вывод, который мы можем сделать из этого, заключается в том, что если мы путешествуем по каждому из векторов по очереди, в любом порядке, который мы выберем (помня, что порядок не важен при добавлении векторов), то мы всегда вернемся в точку, в которой мы начал. Это действительно так, как показано на рисунке ниже. Вы можете поиграть с разным порядком этих векторов, но они всегда будут возвращаться к одной и той же точке. Обратите внимание, что вектор, для которого компоненты x и y равны нулю, называется нулевым вектором (или нулевым вектором ).


    Результатом сложения этих векторов является нулевой вектор (0, 0)


    Сложение векторов работает одинаково для трех измерений (или любого количества измерений, если на то пошло). На диаграмме ниже показаны три вектора: a , b и c . Также показан результат, полученный путем сложения этих векторов. Обратите внимание, что относительное направление осей выбрано несколько произвольно.При работе в среде компьютерной графики принято, что горизонтальная ось соответствует оси x , а вертикальная ось — оси y . Таким образом, ось z является осью, перпендикулярной обеим осям x и y . Положительные значения z выходят за пределы экрана (по направлению к пользователю), а отрицательные значения z выходят за пределы экрана (от пользователя).


    Результат — сумма векторов a , b и c .


    Для тех, кто интересуется такими вещами, приведенная выше диаграмма была построена с использованием GeoGebra 5.0 Beta , которая на момент написания все еще находится в стадии разработки. Это программное обеспечение находится в свободном доступе под Стандартной общественной лицензией GNU (веб-сайт: www.geogebra.org). Векторная арифметика показана ниже и очень похожа на то, что мы уже видели. Единственное существенное отличие состоит в том, что у нас есть один дополнительный компонент (компонент z ), который необходимо включить в наши вычисления. Есть еще одна вещь, которую следует отметить относительно сложения векторов — вы можете только складывать векторы, которые имеют такое же количество компонентов .Вы не можете, например, добавить двумерный вектор к трехмерному вектору.

    a + b + c = -2 +-2 + 3
    36 59 59 59 59 6 -5 4 5
    2 3 1 6

    Студентам было предложено «сложить два вектора A и B, чтобы получить новый вектор…

    Контекст 1

    … являются неотъемлемой частью физики, и даже некоторые ранние концепции механики невозможно изучить без понимания векторов и того, как они складываются и вычитаются. В нескольких предыдущих исследованиях было показано, что студентам часто не хватает способности складывать векторы [1-3], и что это может привести к затруднениям студентов с другими темами физики [4,5]. Вопросы по сложению векторов могут быть графически представлены разными способами. Векторы трансляционно инвариантны и поэтому могут быть изображены в бесконечном множестве расположений.Студентам также могут быть предоставлены различные наглядные пособия, помогающие им складывать или вычитать векторы, включая сетки, координатную ось, углы или величины, помеченные определенными значениями. Кроме того, вопросы могут состоять из любого количества векторов, каждый из которых может иметь любое направление или величину. Прошлые исследования показали, что учащиеся используют множество различных методов для добавления векторов, и что у любого учащегося может быть несколько методов на выбор [2,6]. Цель исследования, представленного в этой статье, — выяснить, существует ли связь между способом представления вопросов о сложении векторов и методами, которые студенты выбирают для сложения векторов вместе.Мы разработали четыре вопроса с графическим векторным сложением. В каждом вопросе добавляются одни и те же два вектора, но с разными графическими представлениями и функциями. Для разработки этих вопросов мы использовали предыдущие исследования [1,2,6], чтобы составить список известных графических методов сложения векторов, используемых студентами. Для каждого метода мы перечислили репрезентативные различия, которые, по нашему предположению, будут иметь наибольший эффект в том, чтобы побудить студентов либо использовать, либо не использовать этот метод. Студенты использовали несколько графических методов сложения векторов, которые мы учли при разработке вопросов [6].Наиболее распространенными из этих методов являются метод «голова к хвосту» и метод коммпона. Менее распространенным, но все же часто используемым методом является метод биссектрисы. В методе «голова к хвосту» ученики располагают векторы в порядке «голова к хвосту», а затем соединяют свободный хвост со свободной головой. В методе компонентов студенты разбивают векторы на их компоненты, а затем складывают компоненты обоих векторов вместе, чтобы получить результирующий вектор. Методы «голова к хвосту» и «компоненты» обычно преподаются как стандартные методы и часто могут привести к правильным ответам.Метод биссектрисы, который не преподается как стандартный метод, имеет множество вариаций. При использовании метода биссектрисы учащиеся определяют направление результирующего, выбирая направление между направлениями двух векторов; затем они делают длину результирующего где-то между половиной длины более длинного вектора и величинами двух векторов, сложенных вместе. Из-за неточности метода биссектрисы это часто может приводить к неправильным ответам. Чтобы иметь возможность приписать конкретное изменение представления к результирующему сдвигу в использовании метода, мы ограничились выбором двух различных изменений представления, которые нужно сделать.Это дало нам сетку 2×2 представлений вопросов, показанную на рис. 1. Мы выдвинули гипотезу о сильном эффекте из-за разного расположения векторов в расположении «голова к хвосту» (выровнены) или «хвост к хвосту» (расходятся). Выравнивая векторы по схеме «голова к хвосту», мы, по сути, выполняем первый шаг метода «голова к хвосту» для студентов; выравнивая их по схеме «хвост к хвосту», мы делаем первый шаг наиболее распространенных версий метода биссектрисы. Это различие в начальном представлении векторной пары должно подтолкнуть учащихся к использованию различных методов сложения.Наше предсказание для второго по эффективности изменения представления заключалось в добавлении или удалении координатной сетки. Нгуен и Мельцер обнаружили, что некоторые студенты, которые могли решить вопрос о сложении векторов с имеющейся сеткой, не могли сделать это без сетки [2]. Наличие сетки дает учащимся номера для работы и координатные оси для описания компонентов. Мы прогнозировали, что использование метода компонентов без сетки будет реже из-за повышенной сложности или неоднозначности использования метода.Вместо этого мы предсказали, что студенты будут использовать методы «голова к хвосту» или «биссектриса», два других широко используемых метода. Данные, представленные в этой статье, взяты из вводного курса физики, основанного на алгебре, в Университете штата Мэн (UMaine). Вопросы задавались студентам на лекции перед пятидневным перерывом в обучении по физике и четырехдневным осенним перерывом в школе, которые происходили между седьмой и восьмой неделями семестра. На последней лекции перед перерывом студентам в случайном порядке задавали вопросы с согласованным или несовпадающим расположением, без сетки.Вопросы с сеткой были случайным образом распределены при чтении в первый день после перерыва. Каждый студент ответил на два вопроса в общей сложности, но только на один (без сетки) в лекции и один (на сетку) при чтении. Причина, по которой каждому студенту было задано по два вопроса, заключалась в том, чтобы увеличить количество ответов, которые мы получим. В нашем анализе данных мы сделали предположение, что учащиеся ответили на вопросы до перерыва (без сетки) и после перерыва (сетка) по существу независимо друг от друга. Вопросы без сетки задавались перед вопросами с сеткой, чтобы учащиеся не могли ссылаться на (предоставленную) сетку в вопросах без сетки.Возможные эффекты грунтования будут обсуждаться позже в статье. Данные были закодированы для методов сложения векторов, которые использовали студенты. Поскольку метод, который учащиеся использовали в своих рисунках, не всегда соответствовал тому, который описан в их письменных объяснениях, их нарисованные объяснения кодировались отдельно от их письменных объяснений. Нарисованные объяснения и письменные объяснения были закодированы в один и тот же набор категорий. Статистический анализ данных проводился с использованием точного критерия Фишера. Каждый из методов сложения графических векторов был протестирован по сравнению с двумя другими методами при изменении расположения и сетки.Вместо критерия хи-квадрат использовался точный критерий Фишера, чтобы избежать проблем с низким количеством клеток. В классе, основанном на алгебре UMaine, изменения в графических представлениях привели к статистически различным распределениям методов ответа. Существовала значительная разница между использованием методов «голова к хвосту» и биссектрисы в объяснениях учащихся [Рис. 2] при сравнении вопросов с согласованными и расходящимися договоренностями. Как и предполагалось, учащиеся с большей вероятностью использовали метод «голова к хвосту», чем метод биссектрисы, в вопросах выровненного расположения.Студенты с одинаковой вероятностью описали метод «голова к хвосту» как метод биссектрисы в вопросах с дивергентным расположением. Эти различия были статистически значимыми (точный критерий Фишера, p <0,05) как для пары вопросов с сеткой, так и для пары вопросов без сетки. Наша гипотеза о том, что расположение векторов в вопросе о графическом сложении векторов влияет на относительное распределение метода «голова к хвосту» и метода биссектрисы, верна только для нарисованных объяснений учащихся, а не для их письменных объяснений.Также наблюдалась значительная разница (точный критерий Фишера, p <0,05) между использованием компонентов и методов биссектрисы в письменных объяснениях студентов [рис. 3] между вопросами с согласованными и расходящимися договоренностями. Эта разница в использовании метода была значимой только для пары вопросов без сетки. Студенты с большей вероятностью объясняли свои рассуждения, описывая метод компонентов для вопросов с выровненным расположением и примерно так же склонны использовать метод компонентов, как и метод биссектрисы для вопросов с расходящимся расположением.Эта разница не та, которую мы предсказывали. Необходима дальнейшая работа, чтобы понять, почему объяснения студентов могут отличаться таким образом, и в более общем плане, почему письменные объяснения студентов не совпадают с нарисованными ими объяснениями. Наша вторая гипотеза о том, что на использование метода компонентов влияет наличие координатных сеток, не была подтверждена ни в нарисованных, ни в написанных ...

    Векторов — Physics LibreTexts

    Вектор — величина с величиной и направление.В этих примечаниях векторная величина будет обозначаться жирным шрифтом (например, вектор электрического поля \ (\ mathbf {E} \)). Графически векторы представлены в виде прямых стрелок. Длина стрелки обычно представляет величину вектора, а стрелка указывает в том же направлении, что и вектор.

    Часто бывает полезно комбинировать векторы путем сложения векторов. Например, если мы ищем полный импульс системы, мы складываем все векторы импульса. Чтобы найти результирующую силу, действующую на объект, мы складываем векторы всех сил, действующих на объект.Чтобы найти полное электрическое поле в определенном месте, мы складываем все векторы всех электрических полей в этом месте вместе. Хотя все эти примеры относятся к разным физическим ситуациям, мы складываем векторы одинаково. Однако экторы v не складывают подобные числа!

    Графическое дополнение

    В таблице ниже показаны два вектора \ (\ mathbf {A} \) и \ (\ mathbf {B} \).

    Чтобы сложить эти векторы, мы соединяем стрелки вверх, чтобы образовать «путь», по которому мы можем следовать, всегда идущий в направлении стрелок.Вектор \ (\ mathbf {A + B} \) — это вектор, который соединяет начало этого пути с концом, как показано ниже.

    Добавление компонентов

    Другой метод сложения векторов — разбиение вектора на компоненты. В то время как векторы не складываются как числа, компоненты вектора складываются. Мы разделим этот вектор на компоненты \ (x \) — и \ (y \) , которые являются наиболее распространенным выбором. Для этого мы должны выяснить, на сколько единиц вектор указывает вправо (\ (+ x \)) и на сколько единиц указывает вектор вверх (\ (+ y \)).Иногда для этого используется сетка, как в примере выше. В этом случае мы можем просто подсчитать количество единиц, но во многих ситуациях нам придется использовать тригонометрию, чтобы разбить вектор на компоненты. В приведенном выше примере

    \ [\ textbf {A} = 6 \ textrm {единиц вправо,} -3 \ textrm {единиц вверх} \]

    \ [\ textbf {B} = 4 \ textrm {единиц вправо, } 4 \ textrm {unit up} \]

    \ [\ textbf {A + B} = 10 \ textrm {units right,} \ mathbf {1} \ textrm {unit up} \]

    Чтобы вычесть \ (\ mathbf {B} \) из \ (\ mathbf {A} \), мы складываем векторов \ (\ mathbf {A} \) и \ (\ mathbf {-B} \).\ (\ mathbf {-B} \) — отрицательное значение для \ (\ mathbf {B} \), определенное как \ (- \ mathbf {B} \ Equiv (-1) \ mathbf {B} \). Графически \ (- \ mathbf {B} \) — это стрелка той же величины, что и \ (\ mathbf {B} \), но указывающая в противоположном направлении.

    \ [\ mathbf {A — B} = \ mathbf {A} + (\ mathbf {-B}) \]

    Применение этого метода к компонентам вектора — допустимый способ математического вычитания векторов. Это дает нам

    \ [\ textbf {A — B} = (6-4) \ textrm {единиц вправо,} (-3-4) \ textrm {единиц вверх} \]

    \ [\ textbf {A — B} = 2 \ textrm {units right,} -7 \ textrm {units up} \]

    Величины \ (\ mathbf {A}, \ mathbf {B}, \) и \ (\ mathbf {A + B} \) обозначаются \ (| \ mathbf {A} |, | \ mathbf {B} |, \) и \ (| \ mathbf {A + B} | \) соответственно.2} \ text {units} = 10.05 \ textrm {units} \]

    В этом случае ясно видно, как величины векторов не складываются как числа, то есть \ (| \ mathbf {A} | + | \ mathbf {B} | \ neq | \ mathbf {A + B} | \). То же самое верно и для вычитания \ (| \ mathbf {A} | — | \ mathbf {B} | \ neq | \ mathbf {A — B} | \).

    Умножение вектора на положительное число изменяет величину вектора, но не меняет направление. Умножение \ (\ mathbf {A} \) на 2 дает нам \ (\ mathbf {2A} \), который указывает в том же направлении, что и \ (\ mathbf {A} \), но в два раза длиннее.

    Умножение вектора на отрицательное число изменяет величину вектора и заставляет его указывать в противоположном направлении. Вектор \ (\ mathbf {-2A} \) вдвое длиннее \ (\ mathbf {A} \) и указывает в противоположном направлении.

    Если мы умножим вектор на число с единицами, последний вектор также будет иметь величину с этими новыми единицами. Рассмотрим это уравнение для силы Лоренца.

    \ [\ mathbf {F} = q \ mathbf {E} \]

    • Единицами \ (\ mathbf {F} \) являются \ (\ text {N} \).Единицами \ (q \) являются \ (\ text {C} \). Единицами \ (\ mathbf {E} \) являются \ (\ dfrac {N} {C} \).
    • Если \ (q \) положительно, \ (\ mathbf {F} \) и \ (\ mathbf {E} \) имеют одинаковое направление .
    • Если \ (q \) отрицательно, \ (\ mathbf {F} \) и \ (\ mathbf {E} \) имеют , противоположное направлению .

    Ученые приняли специальные обозначения, которые помогают рисовать векторы в трех измерениях, когда ваша бумага (или экран компьютера) имеет только два измерения. Векторы, которые указывают в направлении, перпендикулярном странице, представлены этими символами:

    • Символ слева с X представляет вектор, указывающий на страницу.
    • Символ справа с точкой представляет вектор, указывающий за пределы страницы.

    Вопросы?

    Эта страница представляет собой краткий обзор материалов, рассмотренных в Physics 7B. Если вы все еще находите, что векторы сбивают с толку, просмотрите свои заметки 7B или поговорите со своим TA в рабочее время.

    Видео с вопросом: добавление двух векторов, заданных в форме компонента

    Стенограмма видео

    Рассмотрим два вектора: 𝐀 и 𝐁.𝐀 равно отрицательным семи 𝐢 шляпе минус семь 𝐣 шляпе и 𝐁 равно отрицательным шести 𝐢 шляпе минус два 𝐣 шляпе. Вычислите 𝐀 плюс 𝐁.

    Этот вопрос дает нам два вектора в компонентной форме и просит нас вычислить их сумму. Наш первый вектор — это 𝐀, что равняется отрицательным семи шляпе минус семь шляпе. Поскольку 𝐢 hat — единичный вектор в 𝑥-направлении, а 𝐣 hat — единичный вектор в-направлении, это означает, что 𝐀 расширяет отрицательные семь единиц в 𝑥-направлении и отрицательные семь единиц в-направлении.Итак, вектор 𝐀 выглядит так. Наш второй вектор — 𝐁, что равняется отрицательным шести шляпе минус две шляпе. Это означает, что вектор 𝐁 простирается на шесть отрицательных единиц в 𝑥-направлении и на две отрицательные единицы в-направлении. Вот так это выглядит.

    Теперь нам нужно сложить эти два вектора вместе. Вопрос дает нам эти векторы в компонентной форме. И в этом случае самый простой способ сложить эти два вектора — это сделать это алгебраически. Для этого мы отдельно складываем 𝑥-компоненты и 𝑦-компоненты двух векторов.Результат этого, сумма этих двух векторов, называется их равнодействующей. Итак, давайте возьмем наши два вектора 𝐀 и 𝐁 и сложим их 𝑥- и 𝑦-компоненты.

    Если мы сложим-компоненты, чтобы получить 𝑥-компонент нашего результирующего вектора, мы получим отрицательные семь плюс отрицательные шесть. Поскольку это-компонента, мы умножаем это на hat. Тогда, если мы сложим 𝑦-компоненты, мы получим отрицательные семь плюс отрицательные два. И это умножается на шляпы. Последний шаг — оценить эти суммы для 𝑥-компоненты и-компоненты.Для-составляющей сложение отрицательных семи и отрицательных шести дает нам отрицательный результат 13. А для 𝑦-составляющей добавление отрицательных семи и отрицательных двух дает нам отрицательные девять. Итак, мы находим, что сумма или равнодействующая векторов и 𝐁 равна отрицательным 13𝐢 шляпа минус девять 𝐣 шляп.

    Основные сведения о векторах

    Основные сведения о векторах Основные сведения о векторах

    1. Векторы и векторные алгебраические операции

    Скаляр — это величина, которая может быть определена числом. в одиночестве.С другой стороны, вектор должен быть задан как числом и направление. С геометрической точки зрения вектор можно определить как отрезок прямой, имеющий определенное направление и определенную длину. В символом вектора обычно будет выделенная жирным шрифтом буква со стрелкой наверху. Однако в этом документе, чтобы упростить работу с документами в Интернете для построения будет использоваться простая полужирная буква. то есть A . В длина вектора — это его величина и записывается как | A | или когда однозначно, так как сама буква курсивом, т.е., А .

    Векторы следуют алгебре, которая мотивируется их приложениями, и мы будем часто их использовать. Эти алгебраические операции описаны в ваша книга, но они включают:

    правило 1 — Существует нулевой вектор.
    правило 2 — вектор A , умноженный на скаляр m — вектор, неизменный по направлению, но длина изменена на коэффициент м . Правило 3 — Отрицательным элементом вектора является исходный вектор перевернут на 180 градусов ;.
    правило 4 — добавляются два вектора, A и B поместив хвост одного на голову другой (в любом порядке) и определение суммы для быть вектором нарисованным
    от хвоста первого до головы второго.Правило 5 — Вектор B можно вычесть из вектора А добавив -B к A .


    В дополнение к этим правилам, два разных способа умножения векторы полезны. Первый — это скаляр или скалярное произведение векторов A и B и определяется, чтобы привести к скаляру, имеющему Значение

    A точка B = | A || B | cos (альфа)


    где альфа — угол между A и B .Почему эта операция определяется таким образом? Причина в том, что оказывается быть полезным при описании определенных физических величин — например, работы. Второй способ умножения векторов предназначен для получения вектора. В вектор или крест продукт имеет величину

    | A крест B | = | A || B | sin (alpha)


    где альфа снова угол между двумя векторами (меньший из двух возможных углов).Векторное произведение полезно при описании вращательного движения, например. В отличие от скалярного произведения, векторное произведение — это вектор. Направление вектора ( A крест B ) определяется так называемым правилом правой руки. Пальцами правой руки рука указывает в направлении A , пальцы повернуты в вектор B (помните — меньший из двух возможных углов). Тогда вектор ( A крест B ) перпендикулярен обоим A и B и указывает в направлении большого пальца.Вектор, пунктирный в себя дает квадрат длины вектора. Таким образом A точка A = A 2 . Мы также видим, что если A перпендикулярно B , то A точка B = 0. Напротив, вектор перекрещивается сам в себя ( A крест А ) равно 0. По правилу правой руки, хотя скалярное произведение коммутирует, векторных продуктов нет. То есть, хотя A точка B = Б точка A , ( A крест B ) не равно ( B крест А ).Вместо этого ( A крест B ) = — ( B крест A ). В этом обсуждении мы сконцентрируемся на операциях сложения и вычитания. Операция скалярного и векторного умножения будут обсуждаться позже в курсе, но только в той степени, в которой они необходимы.

    2. Векторы и прямоугольные координаты Первый навык, который нам нужен, — это сложение (или вычитание) векторов. алгебраически. С этой целью концепция векторных компонентов дает нам требуются инструменты.Вектор — это чистая сущность, стрелка, если угодно, независимая. любой конкретной системы координат. Но как только мы введем координаты, мы можем конкретно описать вектор , как он появляется в этой системе . Для простоты предположим, что двумерная прямоугольная система определена по осям x и y . Тогда вектор можно определить, если мы знать его длину и угол по отношению к x ось. В качестве альтернативы вектор полностью определен, если мы знаем его комплектующие по осям x и y соответственно.Для вектора A , который имеет длину A и составляет угол альфа по отношению к х оси компоненты называются A x и A y . Они определяются как проекция вектора на каждую ось. Местонахождение вектора не имеет значения. Все векторы одинаковой длины и такая же ориентация в данной системе координат x y имеют одинаковые компоненты и эквивалентны друг другу.
    A x = A cos (альфа); A y = A sin (alpha)


    Таким образом, мы полностью определяем вектор либо написав A в терминах ( A , альфа) или A в терминах из ( A x , A y ).

    Возвращаясь к рис. 1, мы можем нарисовать на рисунке любой прямоугольной системы вообще, затем вычислить компоненты векторов А и B отдельно в нарисованной нами системе.Затем, зная компоненты A и B по отдельности, мы также автоматически узнаем компоненты суммы C = A + B в той же системе, потому что

    C x = A x + B x и C y = A y + B y .


    Алгебра сложения и вычитания векторов становится Очистить. Выбираем систему координат, вычисляем компоненты в ней система и складывать и вычитать компоненты!


    Действие 1. Векторы А и B определены как A = (5,37 °) и B = (10,53 ° ) в
    конкретная система координат x-y .(Предположим, 37 ° , 90 ° и 53 ° определяет треугольник 3-4-5.)
    a) Найдите каждый вектор в форме компонентов в той же системе x-y .
    б) Найдите компоненты векторов C = A + B и D = A — B.
    c) Используйте триггер, чтобы найти векторы C и D в (величина, угол) обозначение.
    3. Расширение до трех измерений; Базовые векторы в декартовой системе координат Координаты При описании движения решения многих проблем становятся просто, если выбрана правильная система координат.Иногда правильная координата система прямоугольная; иногда лучшим выбором может быть полярный или цилиндрический или даже сферический полярный. В основном мы будем иметь дело с прямоугольными или декартовыми система, в которой оси x-y-z все ортогональны (ортогональные перпендикулярно) друг к другу и в порядке, показанном на рисунке 2. В этом системы рисуем в точке P ( x, y, z ) в пространстве вектор A с тремя компонентами A x A y и A z, относящиеся к система координат.

    Как и в двумерном случае, этот вектор полностью указывается, когда мы знаем его компоненты по каждой из трех осей. В чтобы выразить A через выбранную систему координат, мы нужно писать так, чтобы была понятна его векторная природа. К С этой целью мы определяем три единичных векторов (векторов с величиной из 1), каждая из которых параллельна одной из трех осей. В нашей системе x-y-z эти единичные векторы равны i, j и k и параллельны x , л и z соответственно.Теперь, если единичный вектор i умножить скаляром Ax , то Ax i — вектор длины Ax и направлен параллельно оси x . Как следствие, А можно написать

    A = A x i + A y j + A z k ,


    Это удобное обозначение, зная, что сложение и вычитание векторов выполняется с помощью компонентов.Обозначения дают нам алгебраическую форму в котором можно выразить множество векторных операций. Так, например, может быть выражена разница между двумя векторами, A и B как B — A = ( B x — A x ) i + ( B y A y ) j + ( B z A z ) к . Для многих задач полные три измерения могут не понадобиться потому что движение ограничено плоскостью.В этом случае обычная практика — использовать x и y , исключив z .

    Как обсуждалось ранее, общий вектор описывает физический количество в точке пространства и не зависит от какой-либо системы координат. Оси координат можно размещать где угодно и с любой ориентацией. При описании с точки зрения различных декартовых систем, данный вектор может иметь разные выглядящие компоненты, но длина и ориентация вектора остаются одинаковый.



    Действие 2: вернитесь к действию 1 и выразите все результаты в обозначении единичного вектора.

    Задание 3: (Небольшая задача): Учитывая вектор G = i + 2j + k, используйте тригонометрию до
    найдите длину вектора и угол, который вектор образует с относительно оси x .


    4. Вектор положения в декартовых координатах Допустим, мы хотим описать позицию объекта, движущегося в пространстве по пути, описываемому функцией P ( x , y , z ).Мы используем прямоугольную систему, как и раньше, с ортогональными осями x-y-z . В этой системе координат мы рисуем вектор особого вида, называемый позицией вектор с хвостом в начале координат и головой в позиции P ( x , y , z ). Этот вектор, обозначенный как R на рисунке 3, полностью задается следующим образом: компоненты вдоль каждой оси. В чтобы выразить R через выбранную систему координат, мы используйте три единичных вектора i, j и k .Как следствие, R является
    R = R x i + R y j + R z k º x i + y j + z k ,


    Когда объект перемещается в пространстве, вектор положения прослеживает путь. Предположим, что стоимость R на двух разных раз известно. Разница между этими двумя векторами положения составляет

    R 2 — R 1 = дельта R = ( x 2 x 1 ) i + ( y 2 л 1 ) j + ( z 2 z 1 ) k .


    Это последнее выражение представляет собой вектор из ( x 1 , y 1 , z 1 ) к ( x 2 , y 2 , z 2 ) и точно такой же, как векторы A или B , обсуждаемые в разделе 4 . Важное различие между вектором положения R и общий вектор, такой как дельта R , состоит в том, что компоненты R равны x , y и z , тогда как для дельта R компоненты — это дельта x , дельта y и дельта z .Это важно чтобы различать истинные векторы и векторы положения. Настоящий вектор не зависит от системы координат, а только от разницы между один конец вектора и другой. Вектор положения, напротив, делает зависят от системы координат, потому что она используется для определения местоположения относительно заданной контрольной точки.

    Некоторые из предыдущих обсуждений могут показаться вам абстрактными. Не беспокойтесь об этом, потому что постепенно вы научитесь идеи, как вы их используете и как видите, затем используются в лекциях.Вектор нотация — это просто сокращенный способ выражения информации в компактном но точный способ.

    Ответы на занятия

    1. a) A x = 4, A y = 3, B x = 6, B y = 8
    b) C x = 10, C y = 11, D x = -2, D y = -5
    c) C = (14.9,48 ° ), D = (5.4, -68 °)

    2. A = 4 i + 3 j , B = 6 i + 8 j , C = 10 i + 11 j , D = — 2 i — 5 к

    3. Квадрат длины вектора равен G 2 = 1 2 + 2 2 + 1 2 . Таким образом, L = sqrt 6.

    Угол между вектором и осью x равен угол, косинус которого равен G x / G.

    alexxlab

    Добавить комментарий

    Ваш адрес email не будет опубликован. Обязательные поля помечены *